Sie sind auf Seite 1von 74

SHIBIL JALEEL

511231186
Master of Business Administration SEMESTER 1 00094 SPRING 2012

SEMESTER 1 - 511231186

MB0038 Management Process and Organizational Behaviour (Assignment Set- 1) 1. State the characteristics of management.
Management can be characterized as follows: Management is a group activity: - Management is an essential part of group activity. As no individual can satisfy all his desires himself, he units with his fellow- being and works in an organized group to achieve what he cannot achieve individually. Management is goal-oriented: - Management aims to achieve economic and social objective. It exists to achieve some definite goals or objectives. Group efforts in management are always directed toward the achievement of some pre-determined goals. Management is a factor of production: - Management is anuran end in itself but a means to achieve the group objectives. Just as land, labor and capital are factor of production and are essential for the production of goods and services. Management is universal in character: - Management is applicable in all types of organization. Whenever there is human activity, there is management. The basic principle of management are universal application and can be applied in all organization whenever they are business, social, religious, cultural, sport, educational, politics or military. Management is needed at all levels of the organization: - Another important feature of management is that it is needed at all levels of the organization, e.g. top level, middle level and supervisory level. The only difference is of the nature of task and the scope of authority. Management is a distinct process: - Management is a distinct process performed to determine and accomplish started objective by the use of human beings and other resources. It is different from the activities technique and procedures. Management is a social process: - Management is getting thing through others. This involves dealing with people. The efforts of the human beings have to be directed, co-ordinate and regulated by management in order to achieve the desired results. Management is a system of authority: - Since management is a process of directing men to perform a task, authority to accomplish the work from others is implied in the very concept of management. Management cannot perform in the absence of authority. Management is a dynamic function: - Management is a dynamic function and it has to be performed continuously. It is constantly engaged in the molding of the enterprise in an over charging business environment. Management is an art as well as a science: - Management is a science because it has developed certain principle which is of universal application. But the result of management depend upon the personnel skills of managers and in this sense management is an art. Management is a profession: - In the present days, management is recognized as a profession. It has a systematic and specialized body of knowledge consisting of principle, technique and laws and can be taught as a separate discipline or subject.

i.

ii.

iii.

iv.

v.

vi.

vii.

viii.

ix.

x.

xi.

2|P ag e | SPRIN G 2 0 1 2

SEMESTER 1 - 511231186

xii.

Management is intangible: - Management is intangible, i.e. it can be felt in the form of results and not see. E.g. when we are not able to produce the desired quantity, we say it is the result of poor management.

2. What are the 14 principles of management of Henri Fayol?


Management Principles developed by Henri Fayol: DIVISION OF WORK: Work should be divided among individuals and groups to ensure that effort and attention are focused on special portions of the task. Fayol presented work specialization as the best way to use the human resources of the organization. AUTHORITY: The concepts of Authority and responsibility are closely related. Authority was defined by Fayol as the right to give orders and the power to exact obedience. Responsibility involves being accountable, and is therefore naturally associated with authority. Whoever assumes authority also assumes responsibility. DISCIPLINE: A successful organization requires the common effort of workers. Penalties should be applied judiciously to encourage this common effort. UNITY OF COMMAND: Workers should receive orders from only one manager. UNITY OF DIRECTION: The entire organization should be moving towards a common objective in a common direction. SUBORDINATION OF INDIVIDUAL INTERESTS TO THE GENERAL INTERESTS: The interests of one person should not take priority over the interests of the organization as a whole. REMUNERATION: Many variables, such as cost of living, supply of qualified personnel, general business conditions, and success of the business, should be considered in determining a workers rate of pay. CENTRALIZATION: Fayol defined centralization as lowering the importance of the subordinate role. Decentralization is increasing the importance. The degree to which centralization or decentralization should be adopted depends on the specific organization in which the manager is working. SCALAR CHAIN: Managers in hierarchies are part of a chain like authority scale. Each manager, from the first line supervisor to the president, possesses certain amounts of authority. The President possesses the most authority; the first line supervisor the least. Lower level managers should always keep upper level managers informed of their work activities. The existence of a scalar chain and adherence to it are necessary if the organization is to be successful. ORDER: For the sake of efficiency and coordination, all materials and people related to a specific kind of work should be treated as equally as possible. EQUITY: All employees should be treated as equally as possible. STABILITY OF TENURE OF PERSONNEL: Retaining productive employees should always be a high priority of management. Recruitment and Selection Costs, as well as increased product-reject rates are usually associated with hiring new workers. INITIATIVE: Management should take steps to encourage worker initiative, which is defined as new or additional work activity undertaken through self direction.
3|P ag e | SPRIN G 2 0 1 2

SEMESTER 1 - 511231186

ESPIRIT DE CORPS: Management should encourage harmony and general good feelings among employees.

3. Distinguish between internal and external forces of change.


Internal forces for change External forces for change

A general sense that the business could do better Desire to increase profitability Reorganization to increase efficiency Natural ageing and decline in a business (e.g. machinery, products) Conflict between departments The need for greater flexibility in organizational structures Concerns about ineffective communication, de-motivation or poor business relationships

Increased demands for higher quality and levels of customer service Uncertain economic conditions Greater competition Higher cost of inputs Legislation & taxes Political interests Ethics & social values Technological change Globalization Scarcity of natural resources Changing nature and composition of the workforce

4. Define emotional intelligence. Explain Golemans model of emotional intelligence. Emotional intelligence is the ability to identify, assess, and control the emotions of oneself, of others, and of groups. Various models and definitions have been proposed of which the ability and trait EI models are the most widely accepted in the scientific literature. Goleman describes emotional intelligence as "managing feelings so that they are expressed appropriately and effectively, enabling people to work together smoothly toward their common goals." According to Goleman, the four major skills that make up emotional intelligence are: Self-awareness the ability to read one's emotions and recognize their impact while using gut feelings to guide decisions. Self-management involves controlling one's emotions and impulses and adapting to changing circumstances. Social awareness the ability to sense, understands, and reacts to others' emotions while comprehending social networks. Relationship management the ability to inspire, influence, and develop others while managing conflict.
4|P ag e | SPRIN G 2 0 1 2

SEMESTER 1 - 511231186

5. Explain the different leadership styles as per Managerial Leadership Grid Theory.

The model is represented as a grid with concern for production as the x-axis and concern for people as the y-axis; each axis ranges from 1 (Low) to 9 (High). The resulting leadership styles are as follows: The indifferent (previously called impoverished) style (1, 1): evade and elude. In this style, managers have low concern for both people and production. Managers use this style to preserve job and job seniority, protecting themselves by avoiding getting into trouble. The main concern for the manager is not to be held responsible for any mistakes, which results in less innovative decisions. The accommodating (previously, country club) style (1, 9): yield and comply. This style has a high concern for people and a low concern for production. Managers using this style pay much attention to the security and comfort of the employees, in hopes that this will increase performance. The resulting atmosphere is usually friendly, but not necessarily very productive. The dictatorial (previously, produce or perish) style (9, 1): control and dominate. With a high concern for production, and a low concern for people, managers using this style find employee needs unimportant; they provide their employees with money and expect performance in return. Managers using this style also pressure their employees through rules and punishments to achieve the company goals. This dictatorial style is based on Theory X of Douglas McGregor, and is commonly applied by companies on the edge of real or perceived failure. This style is often used in cases of crisis management. The status quo (previously, middle-of-the-road) style (5, 5): balance and compromise. Managers using this style try to balance between company goals and workers' needs. By giving some concern to both people and production, managers who use this style hope to achieve suitable performance but doing so gives away a bit of each concern so that neither production nor people needs are met. The sound (previously, team style) (9, 9): contribute and commit. In this style, high concern is paid both to people and production. As suggested by the propositions of Theory Y, managers choosing to use this style encourage teamwork and commitment among employees. This method relies heavily on making employees feel themselves to be constructive parts of the company. The opportunistic style: exploit and manipulate. Individuals using this style, which was added to the grid theory before 1999, do not have a fixed location on the grid. They adopt whichever behaviour offers the greatest personal benefit. The paternalistic style: prescribe and guide. This style was added to the grid theory before 1999. In The Power to Change, it was redefined to alternate between the (1, 9) and (9, 1) locations on the grid. Managers using this style praise and support, but discourage challenges to their thinking.

5|P ag e | SPRIN G 2 0 1 2

SEMESTER 1 - 511231186

6. Mr. Suresh Kumar is the VP- HR of a leading financial services company. He is


having a meeting with Ms. Rejani Chandran leading HR consultant. Mr. Suresh is concerned about creating an environment that helps in increasing the job satisfaction amongst employees. Assume that you are Ms. Rejani, the HR consultant. What suggestions you will give to Mr. Suresh, for creating an environment that increases job satisfaction?
Measuring Job Satisfaction: Job satisfaction is the sense of fulfillment and pride felt by people who enjoy their work and do it well. For an organization, satisfied work force ensures commitment to high quality performance and increased productivity Job satisfaction helps organizations to reduce complaints and grievances, absenteeism, turnover, and termination. Job satisfaction is also linked to a healthier work force and has been found to be a good indicator of longevity. And although only little correlation has been found between job satisfaction and productivity, it has also been found that satisfying or delighting employees is a prerequisite to satisfying or delighting customers, thus protecting the bottom line (Brown, 1996). The most important factors conductive to job satisfaction are: i) Mentally Challenging Work: Employees tend to prefer jobs that give them opportunities to use their skills and abilities and offer a variety of tasks, freedom and feedback on how well they are doing. Under conditions of moderate challenge, most employees will experience pleasure and satisfaction. ii) Personality-Job Fit: People with personality types congruent with their chosen vocations should find they have the right talents and abilities to meet the demands of their jobs; and because of this success, they have a greater probability of achieving high satisfaction from their work. It is important, therefore to fit personality factors with job profiles. iii) Equitable Rewards: Employees want pay systems and promotion policies that they perceive as being just, unambiguous, and in line with their expectations. When pay is seen as fair based on job demands, individual skill level, and industry pay standards, satisfaction is likely to result. Similarly, employees seek fair promotion policies and practices. Promotions provide opportunities for personal growth, more responsibilities and increased social status. Individuals who perceive that promotion decisions are made in a fair and just manner are likely to experience job satisfaction. iv) Supportive working conditions: Employees prefer physical conditions that are comfortable and facilitate doing a good job. Temperature, light, noise and other environmental factors should not be extreme and provide personal comfort. Further, employees prefer working relatively close to home, in clean and relatively modern facilities and with adequate tools and equipment. v) Supportive Colleagues: Employees have need for social interaction. Therefore, having friendly and supportive co-workers and understanding supervisors leads to increased job satisfaction. Most employees want their immediate supervisor to be understanding and friendly, those who offer praise for good performance, listen to employees opinions and show a personal interest in them.
6|P ag e | SPRIN G 2 0 1 2

SEMESTER 1 - 511231186

vi) Whistle blowing: Whistle-blowers are employees who inform authorities of wrongdoings of their companies or co-workers. Whistle blowing is important because committed organizational members sometimes engage in unethical behaviour in an intense desire to succeed. Organizations can manage whistle blowing by communicating the conditions that are appropriate for the disclosure of wrongdoing. Clearly delineating wrongful behaviour and the appropriate ways to respond are important organizational actions. vii) Social Responsibility: Corporate social responsibility is the obligation of an organization to behave in ethical ways in the social environment in which it operates. Socially responsible actions are expected of organizations. Current concerns include protecting the environment, promoting worker safety, supporting social issues, investing in the community, etc. Managers must encourage both individual ethical behaviour and organizational social responsibility. Job enrichment: It is a deliberate upgrading of responsibility, scope, and challenge in the work itself. Job enrichment usually includes increased responsibility, recognition, and opportunities for growth, learning, and achievement. Large companies that have used job-enrichment programs to increase employee motivation and job satisfaction include, AT&T, IBM, and General Motors (Daft, 1997). Workers role in job satisfaction A worker should also take some responsibility for his or her job satisfaction. Everett (1995) proposed the following questions which employees ask themselves in regard to job satisfaction at the workplace: 1. When have I come closest to expressing my full potential in a work situation? 2. What did it look like? 3. What aspects of the workplace were most supportive? 4. What aspects of the work itself were most satisfying? 5. What did I learn from that experience that could be applied to the present situation? The following suggestions can help a worker find personal job satisfaction: 1. Seek opportunities to demonstrate skills and talents. 2. Develop communication skills. 3. Acquire job related skills and try to implement them. 4. Demonstrate creativity and initiative. 5. Improve team building and leadership skill. 6. Learn to de-stress.

7|P ag e | SPRIN G 2 0 1 2

SEMESTER 1 - 511231186

MB0038 Management Process and Organizational Behaviour (Assignment Set- 2) 1. Explain Sensitivity Training.
This approach evolved from the group dynamics concept of Kurt Lewin and the first sensitivity training session was held in 1946 in State Teachers College, New Britain, USA. Since then, it spread to numerous training centers in USA and other countries. Sensitivity training is a smallgroup interaction process in the unstructured form which requires people to become sensitive to others feelings in order to develop reasonable group activity. The objectives of sensitivity training are as follows: 1. To make participants increasingly aware of, and sensitive to, the emotional reactions and expressions in themselves and others. 2. To increase the ability of participants to perceive, and to learn from, the consequences of their actions through attention to their own and others feelings. 3. To stimulate the clarification and development of personal values and goals consonant with a democratic and scientific approach to problems of personal and social decisions and actions. 4. To develop achievement of behavioral effectiveness in participants. 5. To develop concepts and theoretical framework for linking personal values and goals to actions consistent with these inner factors and situational requirements. Process of Sensitivity Training: Sensitivity training focuses on small group (T-group) with number of members ranging from ten to twelve. T-groups are designed to provide members with experiential learning about group dynamics, leadership and interpersonal relationships. The basic T-group training or sensitivity training is to change the standards, attitudes and behavior of individuals by using psychological techniques and programs. Based on the sources from where these members are drawn, there may be three types of T-group: stranger-lab, cousin-lab, and family-lab. In the stranger-lab, all participants are from different organizations and they are strangers to each other. In cousin-lab, all participants are from the same organization but from different units.

2. Describe the bases of power.


In the context of inter-personal relationship power may be defined as the ability of a person to influence and control behavior of others. Experts have identified different bases or source of power that a person may have. These are:

Coercive power. Coercive power results from a person's ability to punish or withhold rewards. A person who robs you on the street threatening you with a gun is using this type of power. Resource power. A person has resource power when he or she has the discretion to decide the resources available to you. Thus a person in finance department, who can influence the sanction of other employees' expenditure budget can exercise resource power over them.

8|P ag e | SPRIN G 2 0 1 2

SEMESTER 1 - 511231186

Position power. A person has some authority and discretion assigned to him by virtue of his/her position in the organization structure. This is position power. Expert power. A person has expert power by virtue of being recognized as an expert. We accept the advice of doctor, and even allow him/her to operate upon us because we have faith in his expertise. Information power. Information is like resource power. A person with information can disclose the information selectively to people he wants to favour, and in this way exercise influence over them. Association power. People can also exercise power by their relationship and association with others. People tend accept opinions and wishes of people having good relationship them. Personal power. This type of power flows from the persons personal characteristics including looks, personality, and interpersonal skills. This power has a multiplier effect. It helps a person to enhance the effectiveness of all other type of power.

3. What are the hindrances that we face in perception?


Individuals have a tendency to use a number of shortcuts when they judge others. An understanding of these shortcuts can be helpful toward recognizing when they can result in significant distortions. 1. Selective Perception Any characteristic that makes a person, object, or event stand out will increase the probability that it will be perceived. It is impossible for an individual to internalize and assimilate everything that is seen .Only certain stimuli can be taken in selectively. Selectivity works as a shortcut in judging other people by allowing us to speed-read others, but, not without the risk of drawing an inaccurate picture. The tendency to see what we want to see can make us draw unwarranted conclusions from an ambiguous situation. 2. Halo Effect The halo effect occurs when we draw a general impression on the basis of a single characteristic. For example, while appraising the lecturer, students may give prominence to a single trait, such as, enthusiasm and allow their entire evaluation to be tainted by how they judge the instructor on that one trait which stood out prominently in their estimation of that person. Research suggests that it is likely to be most extreme when the traits to be perceived are ambiguous in behavioral terms, when the traits have moral overtones, and when the perceiver is judging traits with which he or she has had limited experience. 3. Contrast Effects Individuals do not evaluate a person in isolation. Their reaction to one person is influenced by other persons they have encountered recently. For example, an interview situation in which one sees a pool of job applicants can distort perception. Distortions in any given candidates evaluation can occur as a result of his or her place in the interview schedule.

9|P ag e | SPRIN G 2 0 1 2

SEMESTER 1 - 511231186

4. Projection This tendency to attribute ones own characteristics to other people which is called projection can distort perceptions made about others. When managers engage in projection, they compromise their ability to respond to individual differences. They tend to see people as more homogeneous than they really are. 5. Stereotyping Stereotypingjudging someone on the basis of our perception of the group to which he or she belongs. Generalization is not without advantages (Hilton & Hippel, 1996). It is a means of simplifying a complex world, and it permits us to maintain consistency. The problem, of course, is when we inaccurately stereotype. In organizations, we frequently hear comments that represent stereotypes based on gender, age, race, ethnicity, and even weight. From a perceptual standpoint, if people expect to see these stereotypes, that is what they will perceive, whether or not they are accurate.6. First-impression error Individuals places a good deal of importance on first impressions. First impressions are lasting impressions. We tend to remember what we perceive first about a person, and sometimes we are quite reluctant to change our initial impressions. First-impression error means the tendency to form lasting opinions about an individual based on initial perceptions. Primacy effects can be particularly dangerous in interviews, given that we form first impressions quickly and that these impressions may be the basis for long-term employment relationships.

4. What are the consequences of conflict in organisations?


As organizations strive to achieve their goals, they are often met with challenges they must overcome as a team. Challenges leave room for conflict between members, other organizations, communities and other parties involved in the organizations mission. While conflict often has a negative connotation, the effects of conflict within an organization can be positive and negative. Mental Health Concerns: Conflict within an organization can cause members to become frustrated if they feel as if theres no solution in sight, or if they feel that their opinions go unrecognized by other group members. As a result, members become stressed, which adversely affects their professional and personal lives. Organization members may have problems sleeping, loss of appetite or overeating, headaches and become unapproachable. In some instances, organization members may avoid meetings to prevent themselves from experiencing stress and stress-related symptoms. Decrease in Productivity: When an organization spends much of its time dealing with conflict, members take time away from focusing on the core goals they are tasked with achieving. Conflict causes members to focus less on the project at hand and more on gossiping about conflict or venting about frustrations. As a result, organizations can lose money, donors and access to essential resources. Members Leave Organization: Organization members who are increasingly frustrated with the level of conflict within an organization may decide to end their membership. This is especially detrimental when members are a part of the executive board or heads of committees. Once
10 | P a g e | S P R I N G 2 0 1 2

SEMESTER 1 - 511231186

members begin to leave, the organization has to recruit new members and appoint acting board members. In extreme cases, where several members leave or an executive board steps down, organizations risk dissolution. Violence: When conflict escalates without mediation, intense situations may arise between organization members. Its unfortunate, but organizational conflicts may cause violence among members, resulting in legal problems for members and possibly the organization. Inspire Creativity: Fortunately, some organization members view conflict as an opportunity for finding creative solutions to solve problems. Conflict can inspire members to brainstorm ideas, while examining problems from various perspectives. Share and Respect Opinions: As organization members work together to solve conflict, they are more willing to share their opinions with the group. Conflict can also cause members to actively listen to each as they work to accomplish the organizations goals. Improve Future Communication: Conflict can bring group members together and help them learn more about each other. From learning each others opinions on topics relevant to the organizations growth to understanding each members preferred communication style, conflict within an organization can give members the tools necessary to easily solve conflicts in the future. Identify New Members: Within organizations members actively participate in each meeting, enjoy serving on multiple committees and have an opinion on each topic the group discusses. There are also members who seemingly contribute little to the group and observe more than talk. Conflict within an organization can inspire typically silent members to step up and demonstrate their leadership skills by offering meaningful solutions to the problem the group is facing.

5. Explain the four processes of Social Learning Theory.


The social learning theory was proposed by Bandura. It recognizes the importance of observing and modeling the behaviors, attitudes, and emotional reactions of others. According to Bandura (1977), most human behavior is learned observationally through modeling: from observing others one forms an idea of how new behaviors are performed, and on later occasions this coded information serves as a guide for action. Social learning theory explains human behavior in terms of continuous reciprocal interaction between cognitive, behavioral, and environmental influences Social learning has four processes: 1. Attention processes People learn from a model only when they recognize and pay attention to its critical features. In order to learn, it is required to pay attention. Anything that detracts the attention is going to have a negative effect on observational learning. If the is model interesting or there is a novel aspect to the situation, it is more likely to dedicate the full attention to learning. 2. Retention processes A models influence will depend on how well the individual remembers the models action after it is no longer readily available. The ability to store information is also an
11 | P a g e | S P R I N G 2 0 1 2

SEMESTER 1 - 511231186

important part of the learning process. Retention can be affected by a number of factors, but the ability to pull up information later and act on it is vital to observational learning. 3. Motor reproduction processes After a person has seen a new behavior by observing the model, the watching must be converted to doing. The ability to store information is also an important part of the learning process. Retention can be affected by a number of factors, but the ability to pull up information later and act on it is vital to observational learning. 4. Reinforcement processes Individuals will be motivated to exhibit the modeled behavior if positive incentives or rewards are provided. Finally, in order for observational learning to be successful, you have to be motivated to imitate the behavior that has been modeled. Reinforcement and punishment play an important role in motivation

6. Ms.Chanchal Das Gupta is a recruitment specialist. For the post of QC Manager, she interviews three candidates. Given below are the physical characteristics of the candidates
Candidate Mr.Ravi Mr.Gineesh Mr.Ramgopal Physical Characteristics Muscular, thick skin, rectangular shaped. Thin, delicate build, large brain, tall. Soft, round shaped, underdeveloped muscles.

From the above descriptions, what personality traits can Ms. Chanchal derive out of the candidates as per Sheldons theory of personality?
Per Sheldon`s theory of personality, below are the traits that Ms. Chanchal can derive: Mr. Ravi represents Mesomorph body type. He is well-proportioned. Psychologically he is Adventurous, Courageous, Indifferent to what others think or want, Assertive/bold, Zest for physical activity, Competitive, with a desire for power/dominance, and a love of risk/chance Mr. Gineesh represents Ectomorph body type. Psychologically he is Self-conscious, Private, Introverted, Inhibited, Socially anxious, Artistic, Intense, Emotionally restrained, Thoughtful Mr. Ramgopal represents Endomorph body type. Psychologically he is Sociable, Fun-loving, Love of food, Tolerant, Even-tempered, Good-humored, Relaxed, with a love of comfort, and has a need for affection.

12 | P a g e | S P R I N G 2 0 1 2

SEMESTER 1 - 511231186

MB0039 Business Communication - Assignment (Set- 1)

1. List the importance of effective communication in the workplace.


Effective communication serves the following specific purposes in an organization:

Greater Awareness of Organizational Goals and Teamwork When there is open communication between superiors, co-workers and subordinates, there is smooth flow of information regarding the goals of the organization. Coordination between the different departments in particular, leads to greater motivation to work together towards achieving a common organizational goal, rather than working in isolation. Better Employer-employee Relationships By listening to employees, showing empathy and giving them the freedom to express their opinions without fear of being repressed, a manager can create a climate of openness that leads to better work relationships. Employees will then feel more comfortable in approaching their superiors and discussing any matter with them. Problem-solving Effective communication can help resolve conflicts between co-workers, work related and performance related problems. Faceto-face communication is especially suited for achieving this task, since it is one to one and highly personalized in nature. Improved Performance Effective communication by managers at the time of appraising the performance of their employees can point out areas for improvement. A constructive review of performance, through which a manager gives positive feedback and counsels the employee, instead of criticizing him for poor performance, can motivate the employee to perform better. Stronger Link between Managers and the External Environment Apart from internal communication within the organization, effective communication by managers with external audiences such as customers, government, bankers, media and suppliers leads to a better rapport with them. A manager will be able to understand the needs of his customers, be aware of the presence of quality suppliers of material, of government regulations and of the expectations of the community at large, only through proper communication.

2. Explain the different aspects of non-verbal communication.


Non-verbal communication, defined as communication without words. It refers to any way of conveying meanings without the use of verbal language. Non-verbal communication is generally unintentional, unlike verbal communication. All of us tend to communicate silently and unknowingly send signals and messages by what we do, apart from what we say. Gestures, facial expressions, posture and the way we dress, are all part of non-verbal communication. Nonverbal communication can have a greater impact than verbal communication, since how you say something is sometimes more important than what you say. Although non-verbal communication can affect both our personal and business relationships, it is particularly important in the workplace. While the spoken or written words may be perfect, the non-verbal aspects could convey the exact opposite meaning. Aspects of non-verbal communication: 1. Kinesics: This is the most often studied and important area of non-verbal communication and refers to body movements of any kind. Different body movements can express inner states of emotion. Facial Expressions can convey feelings of surprise, happiness, anger and sadness. If you
13 | P a g e | S P R I N G 2 0 1 2

SEMESTER 1 - 511231186

meet a long lost friend and say Im very happy to meet you again, but with a sad facial expression, it conveys the exact opposite meaning. Eye Movements, such as wide open pupils express feelings of surprise, excitement or even fear. The importance of eye contact with ones audience was pointed out earlier. Direct eye contact is an indication of intensity and interest, while lack of it can convey feelings of nervousness and guilt. Gestures, such as movement of the hands while giving a lecture or presentation indicates a high level of involvement in what you are saying. On the other hand, shuffling of the feet is a sign of nervousness and speaking with ones hands in ones pockets is considered to be casual or even rude. 2. Proxemics: Proxemics is derived from the word proximity or closeness and is the communication term for personal space and distance. The space and distance which we choose to keep from people is also part of non-verbal communication. Each of us has our own inner and outer circles, which differ for different people. Our inner most circle is an intimate space, into which we generally admit only select people such as family and close friends. Next comes a personal space which might include other friends and colleagues or coworkers. These two spaces involve communication of an informal nature. Most of us also have a social and public space, which includes official or workplace relationships, where the communication is of a more formal nature. In a business context, it is more relevant to understand the concept of fixed space and semi-fixed space. Fixed space means that the physical features of the work environment such as furniture, room size and seating arrangement are permanent. 3. Time Language: This refers to the meaning or importance attached to time and varies between different people. One person may value time more than another. Similarly, time language also varies across cultures. In most western cultures for example, punctuality is considered to be important. Arriving late for a business meeting is inexcusable. In other cultures, it is more relaxed and time is not given that much importance. We convey messages to others through the time we spend on a work related activity or by the importance that we give to time. Arriving early at work or for a job interview shows interest, involvement and seriousness. Spending time with an employee and giving him suggestions on how to improve his performance shows interest and involvement in his career growth. 4. Paralanguage: Para means like or similar to, therefore paralanguage means like language. Of all the forms of non-verbal communication, paralanguage is closest to verbal communication. It refers to the tone of voice with which something is said. In other words, it is how something is said, and not what is said. The tone of voice includes the pitch (high or low pitch), the pace (slow or fast) the emphasis on words and the volume (soft or loud) and can convey different moods and emotions. 5. Physical Context: This refers to the physical environment or surroundings within which we communicate and includes two aspects 1) color and layout and 2) design. Colors are known for their symbolic meaning and have associations with different feelings. For example, colors like black and grey are associated with death, mourning and negative feelings. Yellow and green are associated with more positive feelings.

3. Write short notes on (a) Upward communication (b) Downward communication (c) Horizontal communication
Upward Communication This may be defined as information that flows from subordinates to superiors. Some of the reasons for upward communication include discussing work related problems, giving suggestions for improvement and sharing feelings about the job and co14 | P a g e | S P R I N G 2 0 1 2

SEMESTER 1 - 511231186

workers. This type of communication has both benefits and disadvantages. One of the biggest benefits is problem-solving. Once a subordinate has brought a problem to his superiors notice, chances are that the problem will not recur, since the subordinate learns from his superior how to tackle it the next time. Thus, his ability to solve new problems and therefore his managerial ability, improves. Another benefit that could arise from upward communication is that valuable ideas and suggestions may sometimes come from lower level employees. Therefore organizations should encourage this kind of communication. A third benefit is that employees learn to accept the decisions of management and thereby work as a team. The biggest problem associated with this type of communication is that it may lead to handing down of decisions by superiors. When subordinates frequently seek the superiors guidance, the latter may adopt an authoritarian approach and merely give instructions, disregarding the subordinates opinion completely. Downward Communication This may be defined as information that flows from superiors to subordinates. The most common reasons for downward communication are for giving job instructions, explaining company rules, policies and procedures and giving feedback regarding job performance. A number of studies have indicated that regular downward communication in the form of feedback given to employees is the most important factor affecting job satisfaction. Therefore organizations today are trying to encourage more of this type of communication. There are both benefits and disadvantages associated with this type of communication. Downward communication that provides regular feedback will be beneficial if the feedback or review of performance is constructive. A constructive review is one where a manager counsels an employee, or advises him on how to improve his performance. On the other hand, a destructive review can destroy employee morale and confidence. Regular downward communication also creates a climate of transparency or openness, where information is passed on through official channels, rather than through rumors. Thirdly, downward communication boosts employee morale, since it indicates that management is involved in their progress. The problems with this type of communication are the danger of doing destructive reviews, as mentioned, and that of message overload. This means that superiors many sometimes burden their subordinates with too many instructions, leading to confusion. Horizontal Communication This type of communication is also known as lateral communication. It may be defined as communication that takes place between co-workers in the same department, or in different departments, with different areas of responsibility. The reasons for this type of communication are for coordination of tasks, sharing of information regarding goals of the organization, resolving interpersonal or work related problems and building rapport. The biggest potential benefit of horizontal communication is the sense of teamwork that is created. Regular communication of this type ensures that all co-workers work together towards achieving a common goal in the overall interest of the organization. The biggest potential problem is that conflicts such as ego clashes are bound to arise, when coworkers at the same level communicate on a regular basis.

15 | P a g e | S P R I N G 2 0 1 2

SEMESTER 1 - 511231186

4. Explain the different barriers to listening .List the differences between discriminative listening and comprehension listening.
Listening is not easy and there are a number of obstacles that stand in the way of effective listening, both within and outside the workplace. These barriers may be categorized as follows 1. Physiological Barriers This was discussed earlier under the barriers to communication. Some people may have genuine hearing problems or deficiencies that prevent them from listening properly. Once detected, they can generally be treated. Other people may have difficulty in processing information, or memory related problems which make them poor listeners. Another physiological barrier is rapid thought. Listeners have the ability to process information at the rate of approximately 500 words per minute, whereas speakers talk at around 125 words per minute. Since listeners are left with a lot of spare time, their attention may not be focused on what the speaker is saying, but may wander elsewhere. 2. Physical Barriers These refer to distractions in the environment such as the sound of an air conditioner, cigarette smoke, or an overheated room, which interfere with the listening process. They could also be in the form of information overload. For example, if you are in a meeting with your manager and the phone rings and your mobile beeps at the same time to let you know that you have a message; it is very hard to listen carefully to what is being said. 3. Attitudinal Barriers Pre-occupation with personal or work related problems can make it difficult to focus ones attention completely on what a speaker is saying, even if what is being said is of prime importance. Another common attitudinal barrier is egocentrism, or the belief that you are more knowledgeable than the speaker and that you have nothing new to learn from his ideas. People with this kind of closed minded attitude make very poor listeners. 4. Wrong Assumptions The success of communication depends on both the sender and the receiver, as we have seen in an earlier unit. It is wrong to assume that communication is the sole responsibility of the sender or the speaker and that listener have no role to play. Such an assumption can be a big barrier to listening. For example, a brilliant speech or presentation, however well delivered, is wasted if the receiver is not listening at the other end. Listeners have as much responsibility as speakers to make the communication successful, by paying attention, seeking clarifications and giving feedback. Another wrong assumption is to think that listening is a passive activity, in which a listener merely absorbs the thoughts of the speaker. On the contrary, real listening or active listening is hard work it requires speaking sometimes to ask questions, agree or disagree with the speaker, give feedback, etc. Yet another barrier of this type is to assume that speakers are more powerful than listeners. Speakers are seen as being in command of things, whereas listeners are seen to be weak and lacking authority. According to communication experts however, the reverse is true. Listeners are as important and as powerful as speakers. In fact David J. Schwartz, writer and management professor, emphasizes the importance of listening by saying Big people monopolize the listening. Small people monopolize the talking. 5. Cultural Barriers - Accents can be barriers to listening, since they interfere with the ability to understand the meaning of words that are pronounced differently. The problem of different accents arises not only between cultures, but also within a culture. For example, in a country like
16 | P a g e | S P R I N G 2 0 1 2

SEMESTER 1 - 511231186

India where there is enormous cultural diversity, accents may differ even between different regions and states. Another type of cultural barrier is differing cultural values. The importance attached to listening and speaking differs in western and oriental cultures. Generally, Orientals regard listening and silence as almost a virtue, whereas Westerners attach greater importance to speaking. Therefore this would interfere with the listening process, when two people from these two different cultures communicate. 6. Gender Barriers - Communication research has shown that gender can be a barrier to listening. Studies have revealed that men and women listen very differently and for different purposes. Women are more likely to listen for the emotions behind a speakers words, while men listen more for the facts and the content. Example A salesperson giving a demonstration of a new type of office equipment may be asked by two colleagues if the equipment will work without any problems and respond by saying Sure. A male user may take his answer at face value, whereas a female user may detect some hesitation in his voice. This is because the male user listens for the content of the message, whereas the female user listens for the tone of the message. 7. Lack of Training - Listening is not an inborn skill. People are not born good listeners. They have to develop the art of listening through practice and training. Lack of training in listening skills is an important barrier to listening, especially in the Indian context. Lee Iacocca, former Chairman of the Chrysler Corporation in the US, was one of the first to recognize the need for organized training programs in listening skills. Today, many organizations both in India and abroad incorporate listening skills in their training programs. 8. Bad Listening Habits - Most people are very average listeners who have developed poor listening habits that are hard to shed and that act as barriers to listening. For example, some people have the habit of faking attention or trying to look like a listener, in order to impress the speaker and to assure him that they are paying attention. Others may tend to listen to each and every fact and, as a result, miss out on the main point. Yet another habit is to avoid difficult listening and to tune off deliberately, if the subject is too technical or difficult to understand. Sometimes, the subject itself may be dismissed as uninteresting, because the listener does not want to listen. Discriminative Listening: This is the most basic type of listening, whereby the difference between the sounds is identified. Unless the differences between the sounds are identified, the meaning expressed by such differences cannot be grasped. Once we learn to distinguish between sounds in our own language, we are able to do the same in other languages. One reason why people belonging to one country find it difficult to speak the language of another country is that they find the sounds similar and cannot understand the subtle differences. Likewise, a person who cannot hear the subtleties of emotional variation in another person's voice will be less likely to be able to discern the emotions the other person is experiencing. Listening is a visual as well as auditory act, as we communicate much through body language. We thus also need to be able to discriminate between muscle and skeletal movements that signify different meanings.

17 | P a g e | S P R I N G 2 0 1 2

SEMESTER 1 - 511231186

Comprehension listening - Once we have learnt to discriminate between the different sounds, the next step is to try to comprehend the meaning of these sounds. In order to do this, we require a dictionary of words, along with the rules of grammar and syntax. Apart from the verbal communication, we also need to understand the meaning conveyed by the speakers nonverbal behavior. This can be achieved by closely observing various aspects of the speakers body language and tone of voice. In communication, some words are more important and some less so, and comprehension often benefits from extraction of key facts and items from a long spiel. Comprehension listening is also known as content listening, informative listening and full listening.

5. Discuss the principles of business writing.


The process of good writing involves three basic steps - preparing, writing and editing. Practicing the following 16 principles will help you be a more effective writer. Know your objective: Think before you write. What's your goal? Make sure you fully understand the assignment. Are you writing a one-paragraph executive summary or a five-page report? Try answering this question: What specifically do I want the reader to know, think, or do? Make a list: Write down the ideas or points you want to cover. Why? This helps you get started in identifying the key ideas you want to discuss. If you have trouble getting started, try discussing your ideas with someone else."Kicking an idea around" often helps you clarify your objective and fine-tune what you are trying to accomplish. Organize your ideas: Just as it's difficult to find what you want in a messy, disorganized desk drawer, it's hard to find important ideas in a poorly organized message. Here are a few ways you can organize your ideas: Importance - Begin with the most important piece of information and then move on to the next most important. Chronological order- Describe what happened first, second, third. Problem-Solution - Define the problem, and then describe possible alternatives or the solution you recommend. Question-Answer - State a question and then provide your answer. Organize your ideas so the reader can easily follow your argument or the point you are trying to get across. 4. Back it up: Have an opinion but back it up - support with data. There are a number of ways you can support your ideas, including explanations, examples, facts, personal experiences, stories, statistics, and quotations. It's best to use a combination of approaches to develop and support your ideas. Separate main ideas: Each paragraph should have one main point or idea captured in a topic sentence. The topic sentence is normally the first sentence in the paragraph. Each paragraph should be started by an indentation or by skipping a line.

18 | P a g e | S P R I N G 2 0 1 2

SEMESTER 1 - 511231186

Use bullets or numbers: If you are listing or discussing a number of items, use bullets or number your points like I have done in this paper. Here's an example of using bullets. Join the Business Club to: Increase sales Gain new marketing ideas Make new friends Give back to your profession Write complete sentences: A sentence is about someone doing something - taking action. The someone maybe a manager, employee, customer, etc. The "doing something - taking action" can include mental processes such as thinking, evaluating, and deciding, or physical actions such as writing and talking. A good rule to practice is to have subjects closely followed by their verbs. Use short sentences: Sentences should be a maximum of 12 to 15 words in length. According to the American Press Institute, sentences with 15 or fewer words are understood 90% of the time. Sentences with eight or fewer words are understood 100% of the time. Be precise and accurate: Words like "large," "small," "as soon as possible," "they," "people," "teamwork, "and "customer focus" are vague and imprecise. The reader may interpret these words to mean something different than what you intended. Reduce communication breakdowns by being specific and precise. Define terms as needed. The reader may not understand certain acronyms and abbreviations. Use commas appropriately: Use a comma to separate the elements in a series of three or more items: His favorite colors are red, white, and blue. Use a comma to set off introductory elements: After coffee and donuts, the meeting will begin. Use a comma to separate adjectives: That tall, distinguished, good-looking professor teaches history. Use the correct word: Here are several words that cause confusion. You're is a contraction for "you are" Your means possession, such as "your coat." It's is a contraction for "it is." Its indicates possession. Their means possession/ownership-"their house." There means location. contraction for "they are." They're is a

19 | P a g e | S P R I N G 2 0 1 2

SEMESTER 1 - 511231186

Avoid redundancies: It is a redundancy to use multiple words that mean or say the same thing. For example, consider the following: - My personal beliefs Beliefs are personal, so just state, My beliefs... t - I decided to paint the machine gray in color. Gray is a color, so just state, I decided to paint the machine gray. Numbers: When using numbers in the body of your paper, spell out numbers one through nine, such as "Three men decided" When using numbers 10 or above it's proper to write the number, such as "The report indicated 68 customers" Have a conclusion: Would you really enjoy watching a movie or sporting event that had no conclusion? No. The conclusion ties your points together. The reader wants to know the final score - the bottom line message. Edit your work: Read what you have written several times. first read, focus on organization and sentence structure. Shorten long sentences. Cross out unnecessary words and phrases. Reorganize material as needed. Read it again and make sure commas are used appropriately and that there is punctuation mark at the end of every sentence. Read it a third time and focus on word choice. Are there certain words that are vague or unclear? Replace them with specific words. Read what you have written aloud to yourself or to a friend to see if he or she (and you) can understand it and improve it in any way. A significant part of good writing involves editing. Very few people can sit down and write a perfect paragraph on their first try. It requires multiple rewrites.

6. Explain the advantages of oral communication with the help of suitable example.
Oral communication implies communication through mouth. It includes individuals conversing with each other, be it direct conversation or telephonic conversation. Speeches, presentations, discussions are all forms of oral communication. Oral communication is generally recommended when the communication matter is of temporary kind or where a direct interaction is required. Face to face communication (meetings, lectures, conferences, interviews, etc.) is significant so as to build a rapport and trust. Advantages of Oral Communication There is high level of understanding and transparency in oral communication as it is interpersonal. There is no element of rigidity in oral communication. There is flexibility for allowing changes in the decisions previously taken.
20 | P a g e | S P R I N G 2 0 1 2

SEMESTER 1 - 511231186

The feedback is spontaneous in case of oral communication. Thus, decisions can be made quickly without any delay. Oral communication is not only time saving, but it also saves upon money and efforts. Oral communication is best in case of problem resolution. The conflicts, disputes and many issues/differences can be put to an end by talking them over. Oral communication is an essential for teamwork and group energy. Oral communication promotes a receptive and encouraging morale among organizational employees. Oral communication can be best used to transfer private and confidential information/matter.

21 | P a g e | S P R I N G 2 0 1 2

SEMESTER 1 - 511231186

MB0039 Business Communication - Assignment (Set- 2)

1. List the differences between extensive reading and intensive reading?


Differences between extensive reading and intensive reading Intensive reading It is related to further progress in language learning under the teacher's guidance. It provides a basis for explaining difficulties of structure and for extending knowledge of vocabulary and idioms. It will provide material for developing greater control of the language and speech and writing. Students will study short stories and extracts from novels, chosen for the standard of difficultly of the language and for the interest they hold for this particular group of students. Intensive reading is generally at a slower speed and requires a higher degree of understanding to develop and refine word study skills, enlarge passive vocabulary, reinforce skills related to sentence structure, increase active vocabulary, distinguish among thesis, fact, supportive and non-supportive details, provide socio cultural insights. Extensive reading It develops at the student's own pace according to individual ability. It will be selected at a lower level of difficulty than that for intensive reading. Where frequency word counts are available for the language being learned, extensive reading will conform to a lower frequency word count than intensive reading. Material will be selected whose choice of structure is habitually less complex and whose vocabulary range is less extensive. The purpose of extensive reading is to train the students to read directly and fluently in the target language for enjoyment without the aid of the teacher. Where graded texts are available, structures in texts for extensive reading will be already familiar, and new items of vocabulary will be introduced slowly in such a way that their meaning can be deduced from context or quickly as certained. The student will be encouraged to make intelligent guesses at the meaning of unfamiliar items. Material consists of authentic short stories and plays, or informative or controversial articles from newspapers and magazines. A few adaptations of vocabulary and structure will be made. The style of writing should entail a certain amount of repetition without monotony. Novelties of vocabulary should not coincide with difficulties of structure. It means reading in quantity and in order to gain a general understanding of what is read. It is intended to develop good reading habits, to build up knowledge of vocabulary and structure and to encourage a liking for reading, Increase total comprehension, enable students to achieve independence in basic skill development, acquaint the student with relevant socio-cultural material, and encourage recreational reading.

22 | P a g e | S P R I N G 2 0 1 2

SEMESTER 1 - 511231186

2. Explain the different advantages and disadvantages of intranet.


Advantages and disadvantages of intranet are as follows: Advantages of Intranet There are number of advantages of intranet discussed below: Intranets offering workforce productivity which can help user to find and observe information very fast. User may also use applications according to their roles and tasks. Through web browser a user can get access to entire contents of any website from anywhere or any time. Intranet also increases the ability of employees by performing their job confidently very fast, and accurately. Intranet permits business companies to share out information to employees according to their need or requirements. Employees may also link to appropriate data at their expediency. The best advantage offered by intranet is communications within an organization or business company, landscape or portrait. Intranets are helpful to converse planned initiative that has an international reach all through the organization. The well known examples of transportation are chat, email, and blogs. Actual world example of Intranet is Nestle had a number of food processing plants. The most significant advantage of Intranet is Web publishing which permits burdensome corporate knowledge to be continued and effortlessly access all through the company using Web technologies and hypermedia. The familiar examples of web publishing consist of training, news feed, company polices, documents, and employee manual. Intranet can be accessed general internet standards such as CGI applications, Flash files, and Acrobat files. Each unit can bring up to date the online copy of a document and intranet always provides the most recent version to employees. Intranet is also being used as a platform of mounting and organizing applications across the internet world. Another advantage of Intranet is time saving because there is no need to maintain physical documents such as procedure manual, requisition forms, and internet phone list. Disadvantages of Intranet Intranet has great features for interconnected manners but has some disadvantages too Management does need to stop control of specific information, this problem can be minimized but with appropriate prudence. The other disadvantage of Intranet is security issue
23 | P a g e | S P R I N G 2 0 1 2

SEMESTER 1 - 511231186

Intranet gathered everything in one location which is really good but if it is not prearranged then you will spoil everything. The cost of intranet is very high but has lots of advantages after implementing.

3. List the different principles of business letter writing.


Principles of Writing Business Letters: The format of a business letter is different from other styles. Expect the tone to be less casual and the information kept straight to the point. Keep the intended audience in mind at all times. Like all types of writing, however, planning is a must. First organize your thoughts, and then put pen to paper or fingers to the keyboard

Tone: The tone of a business letter is formal. Avoid casual words and slang. Write the letter as if you applying for a job and not as if you were talking to a friend. Avoid phrases such as, "you know what I mean," "it's cool," and "you know that." Stay away from the light, conversational tone found in emails and messages to friends. This will help you project professionalism in your writing. Intent and Clarity: Jump right in and state the intent of your letter in as few words as possible. Write short and clear sentences. Do not use complicated words when a simpler word will do. Include just enough information so that your message is clear and concise, while still allowing the sentences to flow and maintaining a tactful and polite tone. Pronoun Use: Writing a business letter in the first and second person is acceptable. Using "I" and "you" help to create a connection between the author and reader of the message. When printing your message on paper that includes your business's letterhead, use the pronoun "we." The usage of this word could imply that the opinions or information presented in the letter are those of the entire company rather than just you. Format: The proper format of a business letter includes the date, addresses of the writer and recipient, greeting, body, closing and signature. The recipient may not view your letter as professional if each of these sections is not included.

4. Write short notes on: (a) Corporate identity advertising (b) institutional advertising.
(a) Corporate identity advertising Advertising signatures should be presented in a consistent manner, clearly conveying federal sponsorship of the message. Set out here are guidelines on how government policy on the use of
24 | P a g e | S P R I N G 2 0 1 2

SEMESTER 1 - 511231186

the federal signature and the Canada word mark should be applied in advertising. These guidelines are intended to promote clear and consistent identification of the sponsor, while allowing flexibility on how signatures are being applied. The guidelines were developed to assist all those involved in the planning and preparation of government-sponsored advertising. This section should be used with Chapter 470, Federal Identity Program, and Chapter 480, Government Communications Policy of the Administrative Policy Manual. Official languages requirements with respect to advertising are set out in Chapter 470. Section1. 1, Design, of the FIP Manual contains comprehensive guidelines on the use of symbols and signatures, as well as information on the availability of proofs for reproduction. Guideline examples These guidelines include typical examples to show how the signature and wordmark should be applied in federal advertising. Most of the examples stem from actual advertisements; some of them were modified to reflect the guidelines. Scope Guidelines set out the use of corporate signatures in government-sponsored advertising within Canada. They apply to print advertising, outdoor and transit advertising, television and radio advertising, as well as to paid announcements. Advertising signatures Described below are design criteria that apply to advertising signatures in print advertising, outdoor and transit advertising, and in paid announcements. These criteria pertain to the layout, the relative size and position of the signature and word mark, as well as the use of colour. Federal signature Three aspects determine the design of a signature: layout, type size and typeface. This involves choosing the appropriate layout (e.g. one-, two- or three-line signature), the type size, and the suitable typeface (i.e. Helvetica light, regular or medium). Described below is the effect of these variables on the design of a signature. Layout The signature layout should be chosen on the basis of the signatures length (i.e. number of words) and the space allocated for it in the advertisement. Basically, it is a question of whether a signature should be displayed horizontally, or in a more compact, vertical layout. Where horizontal space is limited, the choice of layout is directly related to the signatures type size. To fit a particular width and to permit the use of a large enough type size, certain signatures may need to be displayed in three or possibly four lines.
25 | P a g e | S P R I N G 2 0 1 2

SEMESTER 1 - 511231186

(b)Institutional advertising Institutional advertising is marketing designed to promote a company rather than a specific good or service. It can be designed to make the public more aware of a company or to improve the reputation and image of an existing company. Depending on the company, this can be a form of brand advertising. Many forms of advertising are about promoting products. This can involve promoting a new product so that the public is aware of its existence, or trying to persuade the public to buy more of an existing product. Institutional advertising instead promotes the company itself. One example would be a grocery chain running advertisements which stressed the general quality or low prices of its food, rather than detailing specific offers it was running. Some forms of institutional advertising are so geared towards promoting a positive image that they effectively discourage sales of a product to some extent. For example, alcohol firms may run commercials warning against excessive drinking or driving while under the influence. Such commercials are usually designed to improve the image of the company, making it seem more trustworthy or responsible. In some cases, institutional advertising is the same thing as brand awareness advertising. This is where the advertising promotes a particular brand rather than the product itself. For example, a banking group might run commercials promoting one of its banks as being dynamic and exciting, while promoting a sister bank as being particularly helpful to customers. In both cases this is different than promoting a specific service, for example by advertising a low rate on loans for new customers. It is also possible for institutional advertising to promote an industry rather than a particular company. This will usually be carried out by an industry association. It happens most often in industries where many of the companies are small firms without the budgets to carry out major advertising, particularly in national media. To give a hypothetical example, most wills prepared by lawyers are done so by small law firms with only a few offices. A trade association for inheritance lawyers could carry out institutional advertising by putting together a television commercial which promotes the importance of getting a will, then lists a website which refers viewers to lawyers in their area. Institutional advertising can cause problems for marketing analysis. Where a commercial is for a specific product, marketers can track how it affects sales and see how effective the advertising was. With institutional advertising, the link between the advertising and the effect on business is much weaker and may take longer to show any effects.

26 | P a g e | S P R I N G 2 0 1 2

SEMESTER 1 - 511231186

5. Discuss the different types of business reports?


Different Types of Business Reports Reports according to function can be subdivided into the following:

Informational reports. Analytical reports Research reports Whereas reports according to formality can be subdivided into the following: Statutory reports Non statutory or voluntary reports The other types of reports are information, analytical, research, statutory, non-statutory, special, and routine reports. Information reports are solely to provide facts without suggestion or personal opinions. Whatever ones findings is that is what reported. These facts are given without personal explanation or, again, any suggestions. Analytical reports are one step further as they contain facts alongside analytical explanation of these facts. They contain a sort of a narration of facts and collected data. They also contain a conclusion or a set of interpretations reached by the writer.

6. List the different steps involved in report preparation


Steps involved in writing a report are as follows:

1. Define the purpose of your report 2. Define the readers of your report 3. Define your sources of data 4. Gather and analyze your data 5. Decide on your recommendations 6. Decide on the key points to include in the report 7. Decide on the best order for your points 8. Decide on the best structure for the report 9. Select data to support your key points 10. Write a draft

27 | P a g e | S P R I N G 2 0 1 2

SEMESTER 1 - 511231186

11. Prepare tables and graphics 12. Edit and proof-read the report 13. Write an executive summary 14. Format the report professionally 15. Make a title page and table of contents

28 | P a g e | S P R I N G 2 0 1 2

SEMESTER 1 - 511231186

MB0040 Statistics for Management - Assignment Set 1 1. What are the functions of Statistics? Distinguish between Primary data and Secondary data.
Statistics is used for various purposes. It is used to simplify mass data and to make comparisons easier. It is also used to bring out trends and tendencies in the data as well as the hidden relations between variables. All this helps to make decision making much easier. Functions of Statistics in detail: 1. Statistics simplifies mass data: The use of statistical concepts helps in simplification of complex data. Using statistical concepts, the managers can make decisions more easily. The statistical methods help in reducing the complexity of the data and consequently in the understanding of any huge mass of data. 2. Statistics makes comparison easier: Without using statistical methods and concepts, collection of data and comparison cannot be done easily. Statistics helps us to compare data collected from different sources. Grand totals, measures of central tendency, measures of dispersion, graphs and diagrams, coefficient of correlation all provide ample scopes for comparison. 3. Statistics brings out trends and tendencies in the data: After data is collected, it is easy to analyse the trend and tendencies in the data by using the various concepts of Statistics. 4. Statistics brings out the hidden relations between variables: Statistical analysis helps in drawing inferences on data. Statistical analysis brings out the hidden relations between variables. 5. Decision making power becomes easier: With the proper application of Statistics and statistical software packages on the collected data, managers can take effective decisions, which can increase the profits in a business. Primary Data & Secondary Data can be distinguished as follows: Primary data is data which has been collected by you, which is more reliable and up to date. Secondary data has been collected from a secondary source (Other people, business etc.) so it may not be valid or up to date. "Primary" and "secondary" are terms used to define data relative to the purpose by which the data were collected originally. "Primary data" are data collected for the need at hand. "Secondary data" are data that were collected for another reason but is being re-purposed to address the need at hand.
29 | P a g e | S P R I N G 2 0 1 2

SEMESTER 1 - 511231186

When describing the expertise of data analysts, it is not uncommon to distinguish between primary and secondary data analytics. Primary data analytics involves the ability to analyze data for the purpose by which it has been collected. Secondary data analytics involves identifying "secondary data sources" to solve a new problem and then the ability to re-purpose that data. Primary data is a data which is created for the first time and there is no previous source available. Secondary data is a readily available data like data from trade directories, statistics from websites etc. In Dissertation Literature review is done through secondary data which includes the contents such as theories, models, compilation, research findings by some other scholar etc.

2. Draw a histogram for the following distribution:


Age No. of people 0-10 5 10-20 10 20-30 15 30-40 8 40-50 2

Histogram
16 14 12 10 8 6 4 2 0 0-10 10-20 20-30 30-40 40-50

3. Find the median value of the following set of values: 45, 32, 31, 46, 40, 28, 27, 37, 36, and 41.

(36+37)/2 = 36.5
30 | P a g e | S P R I N G 2 0 1 2

SEMESTER 1 - 511231186

4. Calculate the standard deviation of the following data:


Marks No. of students 78-80 3 80-82 15 82-84 26 84-86 23 86-88 9 88-90 4

Class Interval 78-80 80-82 82-84 84-86 86-88 88-90

Mid Value X 79 81 83 85 87 89

Frequency f 3 15 26 23 9 4 80

d=x-83 2 -2 -1 0 1 2 3

fd -6 -15 0 23 18 12 32

fd 12 15 0 23 36 36 122

= [122/80 [32/80] ] x (2) = [1.525 - 0.16]x4 =5.46 (mm) Standard Deviation = = 2.336 (mm)
5. An unbiased coin is tossed six times. What is the probability that the tosses will result in: (i) exactly two heads and (ii) at least five heads. The probability of getting exactly 2 heads = .23 The probability of getting at least 5 heads = .11

6.

Explain briefly the types of sampling


Types of samples Probability sampling (Representative samples) Probability samples are selected in such a way as to be representative of the population. They provide the most valid or credible results because they reflect the characteristics of the population from which they are selected (e.g., residents of a particular community, students at an elementary school, etc.). There are two types of probability samples: random and stratified.

31 | P a g e | S P R I N G 2 0 1 2

SEMESTER 1 - 511231186

Random sample The term random has a very precise meaning. Each individual in the population of interest has an equal likelihood of selection. This is a very strict meaning -- you can't just collect responses on the street and have a random sample. The assumption of an equal chance of selection means that sources such as a telephone book or voter registration lists are not adequate for providing a random sample of a community. In both these cases there will be a number of residents whose names are not listed. Telephone surveys get around this problem by random-digit dialing -- but that assumes that everyone in the population has a telephone. The key to random selection is that there is no bias involved in the selection of the sample. Any variation between the sample characteristics and the population characteristics is only a matter of chance. Stratified sample A stratified sample is a mini-reproduction of the population. Before sampling, the population is divided into characteristics of importance for the research. For example, by gender, social class, education level, religion, etc. Then the population is randomly sampled within each category or stratum. If 38% of the population is college-educated, then 38% of the sample is randomly selected from the college-educated population. Stratified samples are as good as or better than random samples, but they require fairly detailed advance knowledge of the population characteristics, and therefore are more difficult to construct. No probability samples (Non-representative samples) As they are not truly representative, non-probability samples are less desirable than probability samples. However, a researcher may not be able to obtain a random or stratified sample, or it may be too expensive. A researcher may not care about generalizing to a larger population. The validity of non-probability samples can be increased by trying to approximate random selection, and by eliminating as many sources of bias as possible. Quota sample The defining characteristic of a quota sample is that the researcher deliberately sets the proportions of levels or strata within the sample. This is generally done to insure the inclusion of a particular segment of the population. The proportions may or may not differ dramatically from the actual proportion in the population. The researcher sets a quota, independent of population characteristics. Purposive sample A purposive sample is a non-representative subset of some larger population, and is constructed to serve a very specific need or purpose. A researcher may have a specific group in mind, such as high level business executives. It may not be possible to specify the population -- they would not all be known, and access will be difficult. The researcher will attempt to zero in on the target group, interviewing whoever is available.
32 | P a g e | S P R I N G 2 0 1 2

SEMESTER 1 - 511231186

A subset of a purposive sample is a snowball sample -- so named because one picks up the sample along the way, analogous to a snowball accumulating snow. A snowball sample is achieved by asking a participant to suggest someone else who might be willing or appropriate for the study. Snowball samples are particularly useful in hard-to-track populations, such as truants, drug users, etc. Convenience sample A convenience sample is a matter of taking what you can get. It is an accidental sample. Although selection may be unguided, it probably is not random, using the correct definition of everyone in the population having an equal chance of being selected. Volunteers would constitute a convenience sample. Non-probability samples are limited with regard to generalization. Because they do not truly represent a population, we cannot make valid inferences about the larger group from which they are drawn. Validity can be increased by approximating random selection as much as possible, and making every attempt to avoid introducing bias into sample selection.

33 | P a g e | S P R I N G 2 0 1 2

SEMESTER 1 - 511231186

MB0040 Statistics for Management Assignment (Set 2)


1. Explain the following terms with respect to Statistics: (i) Sample (ii) Variable (iii) Population.
(i)Sample Sample is a finite subset of a population. A sample is drawn from a population to estimate the characteristics of the population. Sampling is a tool which enables us to draw conclusions about the characteristics of the population.

(ii) Variable A variable in the mathematical sense, i.e. a quantity which may take any one of specified set of values. It is convenient to apply the same word to denote non-measurable characteristics, e.g., sex is a variable in this sense since any human individual may take one of two values, male or female. It is useful, but far from being the general practice, to distinguish between a variable as so defined and a random variable. Variables may be classified into various categories: Categorical variables A categorical variable (also called qualitative variable) is one for which each response can be put into a specific category. These categories must be mutually exclusive and exhaustive. Mutually exclusive means that each possible survey response should belong to only one category, whereas, exhaustive requires that the categories should cover the entire set of possibilities. Categorical variables can be either nominal or ordinal. Nominal variables A nominal variable is one that describes a name or category. Contrary to ordinal variables, there is no 'natural ordering' of the set of possible names or categories. Sex and type of dwelling are examples of nominal variables. In Table 1, the variable "mode of transportation for travel to work" is nominal because it describes the category of transportation. (iii) Population Statistical survey or enquiries deal with studying various characteristics of unit belonging to a group. The group consisting of all the units is called Universe or Population. A statistical population is a set of entities concerning which statistical inferences are to be drawn, often based on a random sample taken from the population. Population is also used to refer to a set of potential measurements or values, including not only cases actually observed but those that are potentially observable.

34 | P a g e | S P R I N G 2 0 1 2

SEMESTER 1 - 511231186

2. What are the types of classification of data?


Classification of Data The process of arranging data into homogenous group or classes according to some common characteristics present in the data is called classification. For Example: The process of sorting letters in a post office, the letters are classified according tothe cities and further arranged according to streets. Bases of Classification: There are four important bases of classification: (1) Qualitative Base (2) Quantitative Base (3)Geographical Base (4) Chronological or Temporal Base (1) Qualitative Base: When the data are classified according to some quality or attributes such as sex, religion, literacy, intelligence etc (2) Quantitative Base: When the data are classified by quantitative characteristics like heights, weights, ages, income etc (3) Geographical Base: When the data are classified by geographical regions or location, like states, provinces, cities, countries etc (4) Chronological or Temporal Base: When the data are classified or arranged by their time of occurrence, such as years, months, weeks, days etc For Example: Time series data. Types of Classification: (1) One -way Classification: If we classify observed data keeping in view single characteristic, this type of classification is known as one-way classification. For Example: The population of world may be classified by religion as Muslim, Christians etc (2) Two -way Classification: If we consider two characteristics at a time in order to classify the observed data then we are doing two way classifications. For Example: The population of world may be classified by Religion and Sex. (3) Multi -way Classification: We may consider more than two characteristics at a time to classify given data or observed data. In this way we deal in multi-way classification. For Example: The population of world may be classified by Religion, Sex and Literacy.

3. Find the (i) arithmetic mean and (ii) range of the following data: 15, 17, 22, 21, 19, 26, 20.

35 | P a g e | S P R I N G 2 0 1 2

SEMESTER 1 - 511231186

Arithmetic Mean = (15+17+22+21+19+26+20)/7 = 20 Range = Highest Value Lowest Value = (26 15) = 11 4. Suppose two houses in a thousand catch fire in a year and there are 2000 houses in a village. What is the probability that: (i) none of the houses catch fire and (ii) At least one house catch fire?
Given the probability of a house catching fire, P = 2/1000 = .002 n = 2000 m=n*p = 2000 * 0.002 = 4 Therefore required probability (i) P ( ) = e- .m / <X = e- 4 = 0.01832 (ii) P(X 1) = 1 P(X = 0) = 1 0.01832 = 0.98168

5. (i) What are the characteristics of Chi-square test? (ii) The data given in the below table shows the production in three shifts and the number of defective goods that turned out in three weeks. Test at 5% level of significance whether the weeks and shifts are independent. Shift 1st Week 2nd Week 3rd Week Total I 15 5 20 40 II 20 10 20 50 III 25 15 20 60 Total 60 30 60 150
The following are the characteristics of Chi-Square Test I. II. III. IV. V. VI. VII. The x test is based on frequencies & not on parameters. It is a non parametric test where no parameters regarding the rigidity if population of populations are required. Additive property is also found in x test The x test is useful to test the hypothesis about the independence of attributes The x test can be used in complex contingency tables. The x test is very widely used for research purposed in behavioural & social sciences including business research. It is defined as x = (O-E)/E where O is the observed frequency & E is the expected frequency.

(ii) The steps followed to calculate x are described below.

36 | P a g e | S P R I N G 2 0 1 2

SEMESTER 1 - 511231186

a) Null hypothesis H: The week & shifts are independent. Alternate hypothesis H : the week & shifts are dependent. b) Level of significance is 5% & D.O.F (3-1)(3-1) = 4 => x tab = 9.49 c) Test Statistics x = (O-E) /E = 3.6459

6. Find Karl Pearsons correlation co-efficient for the data given in the below table: X Y X 20 16 12 8 4 =60 20 22 Y 16 14 4 12 8 Y=60 16 14 12 4 8 12 4 8 XY 440 224 48 96 32 Y = 840

X 400 256 144 64 16 =880

Y 484 196 16 144 64 Y=904

r=

5(840)-(60)(60)________________________ = 0.70 .

37 | P a g e | S P R I N G 2 0 1 2

SEMESTER 1 - 511231186

MB0041 Financial and Management Accounting (Assignment Set 1)


1) The Balanced Score Card is a framework for integrating measures derived from strategy. Take an Indian company which has adopted balance score card successfully and explain how it had derived benefits out of this framework.
Case Study: TATA Motors CVBU (Commercial Vehicle Business Unit) TATA Motors Commercial Vehicle Business Unit enhances balanced scorecard framework.

Tata Motors is the largest and most prominent market leader in the manufacture of commercial business vehicles in India. In the year 2000, its Commercial Vehicles Business Unit (CVBU) suffered its first loss in its more than fifty years history. This loss was massive. It was in the tune of Rs. 108.62 Million. This prompted Tata Motors to take a profound look into itself; to find reason in this debacle. Subsequently, the executive director of CBVU, Mr. Ravi Kant, called for stringent cost cutting across unit operations, supported by more effective formulation and execution of strategy. To augment this process, the management of Tata Motors resolved to adopt the Balanced Scorecard and Performance Framework as the key tool in the endeavor to rebuild the Organisational Performance Chart. The challenge here was to undertake deployment of the Balanced Scorecard across all the functional units and departments of the CBVU. Soon, however, with the process underway, the real problem revealed itself. It turned out that the manual nature of the review procedures of such a huge structure was well neigh impossible, being, at best, extremely difficult to implement and incredibly time consuming. A watertight solution was needed; quickly. After further examination of the situation, a decision was taken to implement a Balanced Scorecard Automation Tool that would centralise, integrate and collate the data, providing rapid review and analytical functionality and presenting a rapid and comprehensive one view picture of organisational performance. Commencing this process, the CVBU management reviewed many solution providers and evaluated each of them upon the basis of a variety of diverse factors. At the end of this exhaustive process, a solution was decided in the form of COVENARK Strategist, a prominent Balanced Score Card Automation Tool developed by mPOWER Information Systems to integrate with the existing ERP and legacy systems with the help of data integration suite. The results were immediate and spectacular. Within two years of this, CVBU had turned over to register a profit of Rs. 107 Million from the loss of Rs. 108.62 Million, accounting for a whopping 60% of TATA Motors inventory turnover. The success path of Balanced Score Card did not stop here. In the beginning CVBU has started the Balanced Scorecard with only Corporate Level Scorecard; at this time they have expanded it to six Hierarchical Levels with three hundred and thirty one Scorecards, additionally looking forward to proliferate it to the lowest level of organizational structure. In this way, balanced scorecard framework played a vital role in the success story of TATA Motors CVBU.

38 | P a g e | S P R I N G 2 0 1 2

SEMESTER 1 - 511231186

2. What is DuPont analysis? Explain all the ratios involved in this analysis. Your answer should be supported with the chart
A method of performance measurement that was started by the DuPont Corporation in the 1920s. With this method, assets are measured at their gross book value rather than at net book value in order to produce a higher return on equity (ROE). It is also known as "DuPont identity". DuPont analysis tells us that ROE is affected by three things: - Operating efficiency, which is measured by profit margin. - Asset use efficiency, which is measured by total asset turnover - Financial leverage, which is measured by the equity multiplier ROE = Profit Margin (Profit/Sales) * Total Asset Turnover (Sales/Assets) * Equity Multiplier (Assets/Equity) Investopedia explains DuPont Analysis It is believed that measuring assets at gross book value removes the incentive to avoid investing in new assets. New asset avoidance can occur as financial accounting depreciation methods artificially produce lower ROEs in the initial years that an asset is placed into service. If ROE is unsatisfactory, the DuPont analysis helps locate the part of the business that is underperforming. The DuPont System expresses the Return on Assets as: ROA = OPM * ATR The Operating Profit Margin Ratio is a measure of operating efficiency and the Asset Turnover Ratio is a measure of asset use efficiency. The DuPont System expresses the Return on Equity as: ROE = (ROA - Interest Expense/Average Assets) * EM The Equity Multiplier is a form of leverage ratio and measures financial efficiency. Figure shows the DuPont Analysis for a farm operation DuPont Analysis for Two Farms Farmer A Farmer B 1. Operating profit margin ratio 0.30 0.12 2. Asset turnover 0.20 0.36 3. ROA (1*2) 0.060 0.043 4. Interest expense to avg. farm assets 0.05 0.03 5. Equity multiplier 2.00 1.50 6. ROE (3-4) * 5 0.02 0.02

39 | P a g e | S P R I N G 2 0 1 2

SEMESTER 1 - 511231186

Farmer A and Farmer B each have a 2 % ROE. The components of the ratios indicate that the sources of the weakness of the farms are different. Farmer A has a stronger profit margin ratio but lower asset turnover compared to Farmer B. Furthermore, Farmer A has a higher leverage ratio than Farmer B. The weak ratios for each farm may be decomposed into components to determine the potential sources of the weakness. To improve asset turnover Farmer A needs to increase production efficiency or price levels or reduce current or noncurrent assets. To improve profit margins, Farmer B needs to increase production efficiency or price levels more than costs or reduce costs more than revenue. The DuPont analysis is an excellent method to determine the strengths and weaknesses of a farm. A low or declining ROE is a signal that there may be a weakness. However, using the analysis you can better determine the source of weakness. Asset management, expense control, production efficiency or marketing could be potential sources of weakness within the farm. Expressing the individual components rather than interpreting ROE it

may identify these weaknesses more readily.

4. Explain any two types of errors that are disclosed by trial balance with examples and rectification entry. Note - Avoid giving examples given in the self- learning material.
1. Errors of omission: If we forget to pass the journal entry of any transaction, both side of trial balance will not be affect with this. So, trial balance will be unsuccessful to disclose these errors. For example, we purchase the goods of Rs. 10,000 but forgot to enter it in journal.
40 | P a g e | S P R I N G 2 0 1 2

SEMESTER 1 - 511231186

2. Errors of commission: These types of errors are happened due to negligence of accountant and cannot be found by making trial balance. Suppose, we sold of Rs. 10,000 goods but recorded in books as Rs. 100. Both account balances in trail balance will not be affected from this. 3. Errors of compensating: Suppose, accountant wrote Rs. 500 less in the debit side of purchase account and same time he also wrote Rs. 500 less in credit side of sale account. Because one mistake is compensated with other error. So, trial balance will not tell such mistakes. 4. Errors of principles: Errors of principles do not affect the trial balance because one side balance will agree with other side balance. Suppose if you have purchased machinery but you debited purchase account in journal entry. It is the mistake of principle but it can not be disclosed by making trial balance. 5. Errors of duplicating: If you pass the journal entry twice, it will not have an effect on the trial balance. So, it will also not be revealed by making trial balance.

5. Distinguish between financial accounting and management accounting

Financial Accounting Reports to those outside the organization owners, lenders, tax authorities and regulators. Emphasis is on summaries of financial consequences of past activities. Objectivity and verifiability of data are emphasized. Precision of information is required. Only summarized data for the entire organization is prepared.

Managerial Accounting Reports to those inside the organization for planning, directing and motivating, controlling and performance evaluation. Emphasis is on decisions affecting the future.

Relevance of items relating to decision making is emphasized. Timeliness of information is required. Detailed segment reports about departments, products, customers, and employees are prepared. Need not follow Generally Accepted Accounting Principles (GAAP). Not mandatory.

Must follow Generally Accepted Accounting Principles (GAAP). Mandatory for external reports.

6. XYZ Ltd provides the following information


Sundry Debtors Cash in hand Cash at Bank 65,000 13,000 15,000 1,05,000 20,000 20,000

41 | P a g e | S P R I N G 2 0 1 2

SEMESTER 1 - 511231186

Bills Receivable Inventory Bills Payables

16,000 90,000 12,000

30,000 84,000 8,000 5,000 58,000 42,000 36,000

Outstanding 6,000 expenses Sundry Creditors 30,000 Bank Overdraft 30,000

Short term Loans 32,000


Prepare a schedule of changes in working capital

42 | P a g e | S P R I N G 2 0 1 2

SEMESTER 1 - 511231186

MB0041 Financial and Management Accounting (Assignment Set 2 )


3. Avon garments Ltd manufactures readymade garments and uses its cut-pieces of cloth to manufacture dolls. The following statement of cost has been prepared.
Particulars Direct material Direct labour Readymade garments Rs. 80,000 13,000 Dolls Rs. 6,000 1,200 2,800 3,000 13,000 12,000 (1,000) Total Rs. 86,000 14,200 19,800 27,000 1,47,000 1,82,000 35,000

Variable 17,000 overheads Fixed overheads 24,000 Total cost Sales Profit (loss) 1,34,000 1,70,000 36,000

The cut-pieces used in dolls have a scrap value of Rs 1,000 if sold in the market. As there is a loss of Rs. 1,000 in the manufacturing of dolls, it is suggested to discontinue their manufacture. Advise the management.

Decision Analysis
Particulars Sales Revenue Production of Dolls Rs. 12,000 Non Production of Dolls 1,000

Scrap Value of Cut Pieces Less advisable costs Variable overheads 2800 Direct Material 6000 Direct Labor 1,200

Differential Revenue

2000

1000

The management is advised that the differential analysis favours continuation of production of dolls.

43 | P a g e | S P R I N G 2 0 1 2

SEMESTER 1 - 511231186

4. Describe the essential features of budgetary control.


Definition: Budgetary Control is defined as "the establishment of budgets, relating the responsibilities of executives to the requirements of a policy, and the continuous comparison of actual with budgeted results either to secure by individual action the objective of that policy or to provide a base for its revision. 2. Salient features: a. Objectives: Determining the objectives to be achieved, over the budget period, and the policy (ies) that might be adopted for the achievement of these ends. b. Activities: Determining the variety of activities that should be undertaken for achievement of the objectives. c. Plans: Drawing up a plan or a scheme of operation in respect of each class of activity, in physical a well as monetary terms for the full budget period and its parts. d. Performance Evaluation: Laying out a system of comparison of actual performance by each person section or department with the relevant budget and determination of causes for the discrepancies, if any. e. Control Action: Ensuring that when the plans are not achieved, corrective actions are taken and when corrective actions are not possible, ensuring that the plans are revised and objective achieved.

5. Briefly describe labor mix variance and yield variance


Labor mix variance is just like material mix variance and it is the part of labor efficiency variance. If you would see any factory, you would see different type of workers. Workers may be skilled, semi skilled and unskilled. Company cost accountant has to make standard quantity of all these type of laborers but actual quantity may also be dependent on the supply of workers. Suppose, we make standard of skilled workers as 40 but actual supplies was of that type worker 30, so we had to appoint semi skilled workers. Due to this, there will happen labor mix variance. There are following main basis of calculating labor mix variance: 1st : If actual composition of laborers is equal to standard composition of laborers Formula of LMV = Standard Cost of Standard Mix of Laborers - Standard Cost of Actual Mix of Laborers 2nd : If actual composition of laborers is not equal to standard composition of laborers Formula of LMV = Total Nos. of Actual labor composition/ total Nos. of standard labor composition X standard cost of revised composition - standard cost of actual composition 3rd If we change standard time composition and then actual time is different from standard time Formula of LMV = Total Nos. of actual labor composition/ total nos. of revised std. labor composition X std. cost of revised std. composition - std. cost of actual composition Yield can be defined as the amount of prime product manufactured from a given amount of materials. The yield variance is the result of obtaining a yield different from the one expected on the basis of input. In sugar refining, a normal loss of yield develops because, on the average it
44 | P a g e | S P R I N G 2 0 1 2

SEMESTER 1 - 511231186

takes approximately 102.5 kilos of sucrose in raw sugar form to produce 100 kilos of sucrose in refined sugars. Part of this sucrose emerges as black strap molasses, but a small percentage is completely lost. In the canning industry, it is customary estimate the expected yield of grades per ton of fruit purchased or delivered to the plant. The actual yield should be compared to the one expected and should be evaluated in terms of cost. If the actual yield deviates from predetermined percentages, cost and profit will differ. Since the final product cost contained not only materials but also labor and factory overhead, a yield variance for labor and factory overhead should be determined when the product is finished. The actual quantities resulting from the processes are multiplied by the standard cost, which includes all three cost elements. A labor yield variance must be looked upon as the result of the quality and /or quantity of the materials handled, while the factory overhead yield variance is due to the greater or smaller number of hours worked. It should be noted that the overhead yield variance may have a significant effect on the amount of over or under absorbed factory overhead.

6. How is standard costing related to budgetary control?


Distinction between Standard Costing and Budgetary Control Although budgetary control and standard costing both are based on some common principles; both are pre-determined, comparison will be made with the actual costs and both system need a revision of the standards or the budget, these two systems have certain differences which are as follows: 1. Budgetary control deals with the operation of a department or the business as a whole in terms of revenue and expenditure. Standard costing is a system of costing which makes a comparison between standard costs of each product or service with its actual cost. 2. Budgetary control covers as a whole in terms of revenue and expenditures such as purchases, sales, production, finance etc. Standard costing is related to a product and its cost only. 3. Budgetary control is applicable to utmost all business organizations. Standard costing is applicable to manufacturing concerns producing standard products and services. 4. Budgetary control is concerned with a specific period and is based on the totals of amounts. Standard costing is concerned with the standard costs, which are worked out generally per unit of production. 5. Budgetary control is not based on standard costing system. Standard costing cannot exist in the absence of a budgetary control system.

45 | P a g e | S P R I N G 2 0 1 2

SEMESTER 1 - 511231186

MB 0042: Managerial Economics - (ASSIGNMENT- Set 1)


1. Define Managerial Economics and explain its main characteristics
Managerial economics is a science that deals with the application of various economic theories, principles, concepts and techniques to business management in order to solve business and management problems. It deals with the practical application of economic theory and methodology to decision-making problems faced by private, public and non-profit making organizations. The same idea has been expressed by Spencer and Seigelman in the following words. Managerial Economics is the integration of economic theory with business practice for the purpose of facilitating decision making and forward planning by the management. According to Mc Nair and Meriam, Managerial economics is the use of economic modes of thought to analyze business situation. Brighman and Pappas define managerial economics as, the application of economic theory and methodology to business administration practice. Joel dean is of the opinion that use of economic analysis in formulating business and management policies is known as managerial economics. Features of managerial Economics 1. It is more realistic, pragmatic and highlights on practical application of various economic theories to solve business and management problems. 2. It is a science of decision-making. It concentrates on decision-making process, decision-models and decision variables and their relationships. 3. It is both conceptual and metrical and it helps the decision-maker by providing measurement of various economic variables and their interrelationships. 4. It uses various macro economic concepts like national income, inflation, deflation, trade cycles etc to understand and adjust its policies to the environment in which the firm operates. 5. It also gives importance to the study of non-economic variables having implications of economic performance of the firm. For example, impact of technology, environmental forces, socio-political and cultural factors etc. 6. It uses the services of many other sister sciences like mathematics, statistics, engineering, accounting, operation research and psychology etc to find solutions to business and management problems.

2. State and explain the law of demand.

In economics, the law of demand is an economic law that states that consumers buy more of a good when its price decreases and less when its price increases (ceteris paribus).The greater the amount to be sold, the smaller the price at which it is offered must be, in order for it to find purchasers. Law of demand states that the amount demanded of a commodity and its price are inversely related, other things remaining constant. That is, if the income of the consumer, prices of the
46 | P a g e | S P R I N G 2 0 1 2

SEMESTER 1 - 511231186

related goods, and tastes and preferences of the consumer remain unchanged, the consumers demand for the good will move opposite to the movement in the price of the good. The negative relation (i.e., higher price attracts lower demand & lower prices encourages high quantity to be bought by the consumers) is based on logic and experience. Mathematically, the inverse relation may be stated with causal relation as:

Where, is the quantity demanded of x goods, f is the function of independent variables contained within the parenthesis, and is the price of x goods? Hence, in the above model, the function (f) is a varying one i.e., the law of demand postulates as the causal factor (independent variable) and is the dependent variable. The two variables move in the opposite direction. When falls rises and the reverse. In regard to the question "by how much will quantity demanded rise?", the law is silent. For example, when for a one-way rail ticket on the Acela Express from Boston's South Station to New York City's Penn Station falls from $111 to $105, ridership may rise from 1625 daily riders to 1825 daily riders or even to just 1626 daily riders. Thus the law of demand merely states the direction in which quantity demanded changes for a given change in price. Moreover, what the law states is hypothetical and not actual.

3. What is Demand Forecasting? Explain in brief various methods of forecasting demand.


Demand forecasting is the activity of estimating the quantity of a product or service that consumers will purchase. Demand forecasting involves techniques including both informal methods, such as educated guesses, and quantitative methods, such as the use of historical sales data or current data from test markets. Demand forecasting may be used in making pricing decisions, in assessing future capacity requirements, or in making decisions on whether to enter a new market. Various methods of forecasting demand 1. Survey of Buyers Intentions or Market Survey Studies. -- Delphi Method 2. The Collective Opinion also called as Sales Force Polling or Expert Opinion polls. 3. Analysis of Time Series and Trend Projections 4. Use of Economic Indicators Regression Analysis and Economic Model Building. 5. Controlled Experiments Test Marketing. 6. Judgmental Approach

1. Survey of Buyers Intentions or Market Survey Studies. -- Delphi Method The most direct method of estimating demand in the short-run is to ask customers what they are planning to buy for the forthcoming time period usually a year. This is very useful when bulk of the sales is to industrial producers. Here the burden of forecasting is shifted to the consumer.
47 | P a g e | S P R I N G 2 0 1 2

SEMESTER 1 - 511231186

In this method, customers may tend to exaggerate their requirements. Customers are numerous, making the method too laborious, impracticable and costly. This method does not expose and measure the variables under the managements control. Delphi Method: This is a variant of the opinion poll or survey method. In Delphi Method, an attempt is made to arrive at a consensus of opinion. The participants are supplied with responses to previous questions from others in the group by a leader. The leader provides each expert with opportunity to react to the information given by others, including reasons advanced, without disclosing the source. 2. The Collective Opinion also called as Sales Force Polling or Expert Opinion polls. Salesmen are required to estimate expected sales in their territories. Salesmen being the closest to the customers, have most intimate feel of the market. The estimates of individual salesmen are consolidated to find out the total estimated sales. These estimates are reviewed to eliminate the bias of optimism or pessimism. Thereafter they are further revised in the light of factors proposed change in prices, product design, advertising budget, expected change in competition, changes in purchasing power, income distribution, employment, population etc. The final forecast will emerge after all these factors are taken into account. The method is known as collective opinion a it takes advantage of the collective wisdom of salesmen, departmental heads like production manager, sales manager, marketing manager, managerial economist and top executives, as well as dealers and distributors. 3. Analysis of Time Series and Trend Projections A firm which has been in existence for some time, will have Accumulated data on sales pertaining to past time periods. Such Data when arranged chronologically yield time series. Time Series of sales represent the past pattern of effective demand for a Particular product. Such data can be presented graphically or in Tabular form. The most popular method of analysis of time series is to project the trend of the time series data. A trend line can be fitted through a series either visually or by means of statistical techniques such as method of least squares. The analyst chooses a plausible algebraic relation (linear, quadratic, logarithmic, etc.) between sales and the independent variable, time. The trend line is then projected into the future by extrapolation. This method is popular because it is simple and inexpensive. The basic assumption is that the past rate of change will continue in the future. Thus the technique yields acceptable results so long as the time series shows a persistent tendency to move in the same direction.

4. Define the term equilibrium. Explain the changes in market equilibrium and effects of shifts in supply and demand.
In economics, economic equilibrium is a state of the world where economic forces are balanced and in the absence of external influences the (equilibrium) values of economic variables will not change. It is the point at which quantity demanded and quantities supplied are equal. Market equilibrium, for example, refers to a condition where a market price is established through competition such that the amount of goods or services sought by buyers is equal to the amount
48 | P a g e | S P R I N G 2 0 1 2

SEMESTER 1 - 511231186

of goods or services produced by sellers. This price is often called the equilibrium price or market clearing price and will tend not to change unless demand or supply changes.
Whenever there is a change in one of the factors of supply or demand, market equilibrium will be affected. Shift in Demand When there is a change of one of the factors of demand- like the price of the product and related goods, consumer preferences, or income- there is a corresponding change in the demand curve. For instance, if someone's income grows, then his demand for goods will increase, shifting his demand curve to the right. This will lead to a higher quantity being consumed at a higher price, ceteris paribus. Conversely, there can be a negative effect that shifts the supply curve to the left where a lower quantity is consumed at a lower price, ceteris paribus. This can occur when the price of substitutes falls or consumers begin to lose their taste for the product. Shift in Supply When there is a change of one of the factors of supply- like changes in the prices of production inputs like labor or capital; a change in production technology and its associated productivity change; or the amount of competition in a specific product market- there is a corresponding change in the supply curve. For example, if worker productivity improves due to some human capital or technology investment, then the costs of production decrease. This exerts a positive effect on the supply curve shifting it right, where the new market equilibrium is at a higher quantity and a lower price, holding everything else constant. There can also be a negative shift that moves the supply curve to the left, with the resulting market clearing price being higher and quantity lower, ceteris paribus. This type of change can occur when the price of an input like labor or raw material jumps. Shifts in Demand and Supply Realistically speaking, ceteris paribus doesn't hold in the real world marketplace as many things are happening at once that either have complimentary or contrary influences upon the market equilibrium. You can't gauge what the new market equilibrium might be as you are not holding everything constant, but two things are being changed simultaneously. To find out what the new market equilibrium is you need detailed information on the magnitude of the supply and demand factor changes and the corresponding shifts in the graph, along with knowledge of the shapes of the curves. Take the market for apples for instance. If both supply and demand increase (on the graph this would be represented by the supply and demand curves both shifting to the right)- if orchard productivity rises while a new medical reports touts the discovery of the newly added health benefits of apples- then the quantity will definitely go up but the new price is indefinite. It could go up if the increase in demand is significant enough, or it could go down if it's not. Similarly, a certain quantity reduction but an uncertain price will pertain when the both demand and supply curves shift to the left. This could happen if the price of apple substitutes like plums drops dramatically, while farm labor becomes much more expensive. There will be certainty about the price, but not the quantity when the supply and demand curves move in opposite directions. For instance, another medical report could come out detailing the unsanitary apple harvesting conditions, shifting demand curve to the left. Simultaneously, genetic engineers have 49 | P a g e | S P R I N G 2 0 1 2

SEMESTER 1 - 511231186 produced an apple that doesn't require as much costly care as before, shifting the supply curve to the right. Price will certainly go down, but the quantity consumed will demand on how relatively large the shifts of each curve are. Similarly, it could be certain that price would go up, but whether quantity consumed would go up or down is uncertain. This could happen if the demand curve shifts to the right while the supply curve shifts to the left- say if everyone's income increases, thereby increasing their consumption of apples and new government regulations curtail farmer's dependence on cheap illegal alien labor. The price of apples would rise, but it would depend on magnitude of the changes in both the supply and demand curves.

5. Explain features of LAC curve with a diagram


Long-run average cost curve (LRAC)

Typical long run average cost curve

The long-run average cost curve depicts the cost per unit of output in the long runthat is, when all productive inputs' usage levels can be varied. All points on the line represent least-cost factor combinations; points above the line are attainable but unwise, while points below are unattainable given present factors of production. The behavioral assumption underlying the curve is that the producer will select the combination of inputs that will produce a given output at the lowest possible cost. Given that LRAC is an average quantity, one must not confuse it with the long-run marginal cost curve, which is the cost of one more unit. The LRAC curve is created as an envelope of an infinite number of short-run average total cost curves, each based on a particular fixed level of capital usage.The typical LRAC curve is U-shaped, reflecting increasing returns of scale where negatively-sloped, constant returns to scale where horizontal and decreasing returns (due to increases in factor prices) where positively sloped In a long-run perfectly competitive environment, the equilibrium level of output corresponds to the minimum efficient scale, marked as Q2 in the diagram. This is due to the zero-profit requirement of a perfectly competitive equilibrium. This result, which implies production is at a level corresponding to the lowest possible average cost, does not imply that production levels other than that at the minimum point are not efficient. All points along the LRAC are productively efficient, by definition, but not all are equilibrium points in a long-run perfectly competitive environment. In some industries, the bottom of the LRAC curve is large in comparison to market size (that is to say, for all intents and purposes, it is always declining and economies of scale exist indefinitely).
50 | P a g e | S P R I N G 2 0 1 2

SEMESTER 1 - 511231186

This means that the largest firm tends to have a cost advantage, and the industry tends naturally to become a monopoly, and hence is called a natural monopoly. Natural monopolies tend to exist in industries with high capital costs in relation to variable costs, such as water supply and electricity supply.

6. Explain cost output relationship with reference to a. Total fixed cost and output b. Total variable cost and output c. .Total cost and output

51 | P a g e | S P R I N G 2 0 1 2

SEMESTER 1 - 511231186

MB 0042: Managerial Economics (ASSIGNMENT- Set 2)


1. Explain the relationship between revenue concepts and price elasticity of demand.
Price elasticity of demand is an important topic in economics. It relates to the key concepts of supply and demand. When something changes in a market -- price, for example -- elasticity tells us how much other things will change. The relationship between price elasticity and total revenue is critical, as it helps to inform management when making pricing decisions for a good or a service. Price Elasticity Price elasticity tells how much of an impact a change in price will have on the consumers' willingness to buy that item. If the price rises, the law of demand states that the quantity demanded of that item will decrease. Price elasticity of demand tells you how much the quantity demanded decreases. Elastic demand means that the consumers of that good or service are highly sensitive to changes in price. Usually, a good which is not a necessity or has numerous substitutes has elastic demand. Inelastic demand means that the consumers of that good are not highly sensitive to price changes. If the price of an inelastic good, say cigarettes, rises by 10 percent, maybe sales will only decrease by 1 percent. Consumers will still buy that good, typically because it is essential or has no substitutes. How to Calculate Price Elasticity The price elasticity of a good or service is calculated as the percent change in the quantity demanded of a good divided by the percent change in the price for that good. A price elasticity greater than one indicates that the good is elastic, that quantity demanded is highly sensitive to changes in price. For example, a 1-percent change in the price of chicken might cause a 5percent decrease in sales. A price elasticity less than one tells you that your good is inelastic. In this case, price changes will have a small impact on quantity demanded. If a good/service has a price elasticity of one, also called unitary elasticity, this means that a 5-percent change in price will result in a 5-percent change in quantity demanded. Total Revenue Total revenue is calculated as the quantity of a good sold multiplied by its price. It is a measure of how much money a company makes from selling its product, before any costs are considered. Obviously, the goal of a company is to maximize profits, and one way to do this is by increasing total revenue. The company can increase its total revenue by selling more items or by raising the price. Why Elasticity Matters Price elasticity of demand and total revenue are closely interrelated because they deal with the same two variables, P and Q. If your product has elastic demand, you can increase your revenue by decreasing the price of that good. P will decrease, but Q will increase at a greater rate, thus increasing total revenue. If the product is inelastic, then you can actually raise prices, sell slightly less of that item but make higher revenue. As a result, it is important for management to know whether its product has inelastic or elastic demand.

52 | P a g e | S P R I N G 2 0 1 2

SEMESTER 1 - 511231186

2. Explain the emergence of Consumers surplus with their practical application.


An economic measure of consumer satisfaction, which is calculated by analyzing the difference between what consumers are willing to pay for a good or service relative to its market price. A consumer surplus occurs when the consumer is willing to pay more for a given product than the current market price. The importance of consumer's surplus is that it provides a monetary measure of the benefit that a consumer derives from the supply of a product given the terms on which it is made available. It seems therefore to offer the possibility of assessing the net effect on welfare of policies that alter the terms on which different products are supplied. Economists have used the concept to argue that some systems of taxation are worse than others because they lead to a greater loss of consumer's surplus. It has also been proposed that in decreasing cost industries in which consumers' expenditures for a product would not cover total costs if the product were sold on the market at a uniform price or, in most modern formulations, at a price equal to marginal costthe state should make possible production of the product by means of a subsidy when the gain in consumer's surplus would justify this. In effect, what consumers would be willing to pay but do not (or, more exactly, the state's estimate of this) should be treated as an auxiliary factor in addition to what they do pay, in determining production, because it indicates as much as what they do pay the worth of the product to them. Consumers always like to feel like they are getting a good deal on the goods and services they buy and consumer surplus is simply an economic measure of this satisfaction. For example, assume a consumer goes out shopping for a CD player and he or she is willing to spend $250. When this individual finds that the player is on sale for $150, economists would say that this person has a consumer surplus of $100.

3. What is monetary policy? Explain the general objectives of monetary policy.


"A policy employing the central banks control of the supply of money as an instrument for achieving the objectives of general economic policy is a monetary policy." "A policy which influences the public stock of money substitute of public demand for such assets of both that is policy which influences public liquidity position is known as a monetary policy." Rapid Economic Growth: It is the most important objective of a monetary policy. The monetary policy can influence economic growth by controlling real interest rate and its resultant impact on the investment. If the RBI opts for a cheap or easy credit policy by reducing interest rates, the investment level in the economy can be encouraged. This increased investment can speed up economic growth. Faster economic growth is possible if the monetary policy succeeds in maintaining income and price stability. Price Stability: All the economics suffer from inflation and deflation. It can also be called as Price Instability. Both inflations are harmful to the economy. Thus, the monetary policy having an objective of price stability tries to keep the value of money stable. It helps in reducing the income and wealth inequalities. When the economy suffers from recession the monetary policy should be an 'easy money policy' but when there is inflationary situation there should be a 'dear money policy'.
53 | P a g e | S P R I N G 2 0 1 2

SEMESTER 1 - 511231186

Exchange Rate Stability: Exchange rate is the price of a home currency expressed in terms of any foreign currency. If this exchange rate is very volatile leading to frequent ups and downs in the exchange rate, the international community might lose confidence in our economy. The monetary policy aims at maintaining the relative stability in the exchange rate. The RBI by altering the foreign exchange reserves tries to influence the demand for foreign exchange and tries to maintain the exchange rate stability. Balance of Payments (BOP) Equilibrium: Many developing countries like India suffers from the Disequilibrium in the BOP. The Reserve Bank of India through its monetary policy tries to maintain equilibrium in the balance of payments. The BOP has two aspects i.e. the 'BOP Surplus' and the 'BOP Deficit'. The former reflects an excess money supply in the domestic economy, while the later stands for stringency of money. If the monetary policy succeeds in maintaining monetary equilibrium, then the BOP equilibrium can be achieved. Full Employment: The concept of full employment was much discussed after Keynes's publication of the "General Theory" in 1936. It refers to absence of involuntary unemployment. In simple words 'Full Employment' stands for a situation in which everybody who wants jobs get jobs. However it does not mean that there is a Zero unemployment. In that senses the full employment is never full. Monetary policy can be used for achieving full employment. If the monetary policy is expansionary then credit supply can be encouraged. It could help in creating more jobs in different sector of the economy. Neutrality of Money: Economist such as Wicksted, Robertson has always considered money as a passive factor. According to them, money should play only a role of medium of exchange and not more than that. Therefore, the monetary policy should regulate the supply of money. The change in money supply creates monetary disequilibrium. Thus monetary policy has to regulate the supply of money and neutralize the effect of money expansion. However this objective of a monetary policy is always criticized on the ground that if money supply is kept constant then it would be difficult to attain price stability. Equal Income Distribution: Many economists used to justify the role of the fiscal policy is maintaining economic equality. However in recent years economists have given the opinion that the monetary policy can help and play a supplementary role in attainting an economic equality. Monetary policy can make special provisions for the neglect supply such as agriculture, smallscale industries, village industries, etc. and provide them with cheaper credit for longer term. This can prove fruitful for these sectors to come up. Thus in recent period, monetary policy can help in reducing economic inequalities among different sections of society.

4. What is a business cycle? Describe the different phases of a business cycle.


The business cycle describes the phases of growth and decline in an economy. The goal of economic policy is to keep the economy in a healthy growth rate fast enough to create jobs for everyone who wants one, but slow enough to avoid inflation. Unfortunately, life is not so simple. Many factors can cause an economy to spin out of control, or settle into depression. The most important, over-riding factor is confidence of investors, consumers, businesses and politicians. The economy grows when there is confidence in the future and in policymakers, and does the opposite when confidence drops.

54 | P a g e | S P R I N G 2 0 1 2

SEMESTER 1 - 511231186

The phases of the Business Cycle There are four stages that describe the business cycle. At any point in time you are in one of these stages: Contraction When the economy starts slowing down. Trough When the economy hits bottom, usually in a recession. Expansion When the economy starts growing again. Peak When the economy is in a state of irrational exuberance.

5. What is inflation? Explain the causes of inflation.


Inflation is the long term rise in the prices of goods and services caused by the devaluation of currency. While there are advantages to inflation which I will discuss later in this article, I want to first focus on some of the negative aspects of inflation. Inflationary problems arise when we experience unexpected inflation which is not adequately matched by a rise in peoples incomes. If incomes do not increase along with the prices of goods, everyones purchasing power has been effectively reduced, which can in turn lead to a slowing or stagnant economy. Moreover, excessive inflation can also wreak havoc on retirement savings as it reduces the purchasing power of the money that savers and investors have squirreled away. Causes of Inflation 1. The Money Supply Inflation is primarily caused by an increase in the money supply that outpaces economic growth. Ever since industrialized nations moved away from the gold standard during the past century, the value of money is determined by the amount of currency that is in circulation and the publics perception of the value of that money. When the Federal Reserve decides to put more money into circulation at a rate higher than the economys growth rate, the value of money can fall because of the changing public perception of the value of the underlying currency. As a result, this devaluation will force prices to rise due to the fact that each unit of currency is now worth less. One way of looking at the money supply effect on inflation is the same way collectors value items. The rarer a specific item is, the more valuable it must be. The same logic works for currency; the less currency there is in the money supply, the more valuable that currency will be. When a government decides to print new currency, they essentially water down the value of the money already in circulation. A more macroeconomic way of looking at the negative effects of an increased money supply is that there will be more dollars chasing the same amount of goods in an economy, which will inevitably lead to increased demand and therefore higher prices. 2. The National Debt We all know that high national debt in the U.S. is a bad thing, but did you know that it can actually drive inflation to higher levels over time? The reason for this is that as a countrys debt increases, the government has two options: they can either raise taxes or print more money to pay off the debt. A rise in taxes will cause businesses to react by raising their prices to offset the increased corporate tax rate. Alternatively, should the government choose the latter option, printing more money will lead directly to an increase in the money supply, which will in turn lead to the devaluation of the currency and increased prices (as discussed above).
55 | P a g e | S P R I N G 2 0 1 2

SEMESTER 1 - 511231186

3. Demand-Pull Effect The demand-pull effect states that as wages increase within an economic system (often the case in a growing economy with low unemployment), people will have more money to spend on consumer goods. This increase in liquidity and demand for consumer goods results in an increase in demand for products. As a result of the increased demand, companies will raise prices to the level the consumer will bear in order to balance supply and demand. An example would be a huge increase in consumer demand for a product or service that the public determines to be cheap. For instance, when hourly wages increase, many people may determine to undertake home improvement projects. This increased demand for home improvement goods and services will result in price increases by house-painters, electricians, and other general contractors in order to offset the increased demand. This will in turn drive up prices across the board. 4. Cost-Push Effect Another factor in driving up prices of consumer goods and services is explained by an economic theory known as the cost-push effect. Essentially, this theory states that when companies are faced with increased input costs like raw goods and materials or wages, they will preserve their profitability by passing this increased cost of production onto the consumer in the form of higher prices. A simple example would be an increase in milk prices, which would undoubtedly drive up the price of a cappuccino at your local Starbucks since each cup of coffee is now more expensive for Starbucks to make. 5. Exchange Rates Inflation can be made worse by our increasing exposure to foreign marketplaces. In America, we function on a basis of the value of the dollar. On a day-to-day basis, we as consumers may not care what the exchange rates between our foreign trade partners are, but in an increasingly global economy, exchange rates are one of the most important factors in determining our rate of inflation. When the exchange rate suffers such that the U.S. currency has become less valuable relative to foreign currency, this makes foreign commodities and goods more expensive to American consumers while simultaneously making U.S. goods, services, and exports cheaper to consumers overseas. This exchange rate differential between our economy and that of our trade partners can stimulate the sales and profitability of American corporations by increasing their profitability and competitiveness in overseas markets. But it also has the simultaneous effect of making imported goods (which make up the majority of consumer products in America), more expensive to consumers in the United States.

6. Write short notes on the following: a. Monopoly b. Oligopoly


A monopoly exists when a specific person or enterprise is the only supplier of a particular commodity. Monopolies are characterized by a lack of economic competition to produce the
56 | P a g e | S P R I N G 2 0 1 2

SEMESTER 1 - 511231186

good or service and a lack of viable substitute goods. The verb "monopolize" refers to the process by which a company gains the ability to raise prices or exclude competitors. In economics, a monopoly is a single seller. In law, a monopoly is business entity that has significant market power, that is, the power, to charge high prices. Although monopolies may be big businesses, size is not a characteristic of a monopoly. A small business may still have the power to raise prices in a small industry (or market). When not coerced legally to do otherwise, monopolies typically maximize their profit by producing fewer goods and selling them at higher prices than would be the case for perfect competition. Sometimes governments decide legally that a given company is a monopoly that doesn't serve the best interests of the market and/or consumers. Governments may force such companies to divide into smaller independent corporations. Monopolies can be established by a government, form naturally, or form by mergers. A monopoly is said to be coercive when the monopoly actively prohibits competitors by using practices (such as underselling) which derive from its market or political influence (see Chainstore paradox). There is often debate of whether market restrictions are in the best longterm interest of present and future consumers. Oligopoly is a market structure characterized by a small number of relatively large firms that dominate an industry. The market can be dominated by as few as two firms or as many as twenty, and still be considered oligopoly. With fewer than two firms, the industry is monopoly. As the number of firms increase (but with no exact number) oligopoly becomes monopolistic competition. Because an oligopolistic firm is relatively large compared to the overall market, it has a substantial degree of market control. It does not have the total control over the supply side as exhibited by monopoly, but its capital is significantly greater than that of a monopolistically competitive firm. Relative size and extent of market control means that interdependence among firms in an industry is a key feature of oligopoly. The actions of one firm depend on and influence the actions of another. Such interdependence creates a number of interesting economic issues. One is the tendency for competing oligopolistic firms to turn into cooperating oligopolistic firms. When they do, inefficiency worsens, and they tend to come under the scrutiny of government. Alternatively, oligopolistic firms tend to be a prime source of innovations, innovations that promote technological advances and economic growth. Like much of the imperfection that makes up the real world, there is both good and bad with oligopoly. The challenge in economics is, of course, to promote the good and limit the bad. Characteristics The three most important characteristics of oligopoly are: (1) an industry dominated by a small number of large firms, (2) firms sells either identical or differentiated products, and (3) the industry has significant barriers to entry.

57 | P a g e | S P R I N G 2 0 1 2

SEMESTER 1 - 511231186

Although oligopolistic industries tend to be diverse, they also tend to exhibit several behavioral tendencies: (1) interdependence, (2) rigid prices, (3) nonprice competition, (4) mergers, and (5) collusion. Interdependence: Each oligopolistic firm keeps a close eye on the activities of other firms in the industry. Decisions made by one firm invariably affect others and are invariably affected by others. Competition among interdependent oligopoly firms is comparable to a game or an athletic contest. One team's success depends not only on its own actions but on the actions of its competitor. Oligopolistic firms engage in competition among the few. Rigid Prices: Many oligopolistic industries (not all, but many) tend to keep prices relatively constant, preferring to compete in ways that do not involve changing the price. The prime reason for rigid prices is that competitors are likely to match price decreases, but not price increases. As such, a firm has little to gain from changing prices. Nonprice Competition: Because oligopolistic firms have little to gain through price competition, they generally rely on nonprice methods of competition. Three of the more common methods of nonprice competition are: (a) advertising, (b) product differentiation, and (c) barriers to entry. The goal for most oligopolistic firms is to attract buyers and increase market share, while holding the line on price. Mergers: Oligopolistic firms perpetually balance competition against cooperation. One way to pursue cooperation is through merger--legally combining two separate firms into a single firm. Because oligopolistic industries have a small number of firms, the incentive to merge is quite high. Doing so then gives the resulting firm greater market control. Collusion: Another common method of cooperation is through collusion--two or more firms that secretly agree to control prices, production, or other aspects of the market. When done right, collusion means that the firms behave as if they are one firm, a monopoly. As such they can set a monopoly price, produce a monopoly quantity, and allocate resources as inefficiently as a monopoly. A formal method of collusion, usually found among international produces is a cartel.

58 | P a g e | S P R I N G 2 0 1 2

SEMESTER 1 - 511231186

MB0043 Human Resource Management - (Assignment Set- 1)


1. Explain the need for human resource planning.
Human resource or manpower planning is the process by which a management determines how an organization should move from its current manpower position to its desired manpower position. Through planning, a management strives to have the right number and the right kind of people at the right places, at the right time, to do things which result in both the organization and the individual receiving the maximum long-range benefit. The organizations business plan to invest in a particular product/market or a service will drive the HRP activity towards hiring to meet the business need. In the event an organization is divesting or shutting down a particular business unit or a manufacturing division the HRP activities would focus on the redeployment of the workforce that will be rendered unemployed as a result of the business decision. An organization wanting to retain its current market share and revenue projection at status quo would be supported by HRP activities that are limited to only filling positions falling vacant due to natural organizational attrition. Hence the HRP focus in a organizations is closely linked to the business plan and acts as a bridge between what an organization wishes to achieve and how it will go about achieving it w.r.t. the human resources requirements. The scope of HRP is futuristic in nature and usually runs parallel to the annual business planning exercise. It commences prior to the start of the companys new financial year. For example if the business year for a company runs April to March. The business planning and the HR planning activities for the New Year beginning in April is usually completed and in place by the 1st week of April. Once the HR plan is in place it is broken up into a quarterly or even a monthly plan which then is the input for the recruitment team to go and hire accordingly. Need for Human Resource Planning: Human Resource Planning is a mandatory part of every organizations annual planning process. Every organization that plans for its business goals for the year also plans for how it will go about achieving them, and therein the planning for the human resource: 1. To carry on its work, each organization needs competent staff with the necessary qualifications, skills, knowledge, work experience and aptitude for work. 2. Since employees exit and organization both naturally (as a result of superannuation) and unnaturally (as a result of resignation), there is an on-going need for hiring replacement staff to augment employee exit. Otherwise, work would be impacted. 3. In order to meet the need for more employees due to organizational growth and expansion, this in turn calls for larger quantities of the same goods and services as well as new goods. This growth could be rapid or gradual depending on the nature of the business, its competitors, its position in the market and the general economy.

59 | P a g e | S P R I N G 2 0 1 2

SEMESTER 1 - 511231186

4. Often organizations might need to replace the nature of the present workforce as a result of its changing needs, therefore the need to hire new set of employees. To meet the challenge of the changed needs of technology / product/service innovation the existing employees need to be trained or new skill sets induced into the organization. 5. Manpower planning is also needed in order to identify an organizations need to reduce its workforce. In situations where the organization is faced with severe revenue and growth limitations it might need to plan well to manage how it will reduce its workforce. Options such as redeployment and outplacement can be planned for and executed properly.

2. What are the objectives of job evaluation?


The decision to measure or rate jobs should only be made with the intent to reach certain objectives which are important to both management and the worker. Although there are many byproducts of job evaluation, the purpose is to work towards a solution of the many wage and salary administrative problems which confront the industry. The below mentioned are some of the important objectives of a job evaluation program: 1. Establishment of sound wage differentials between jobs. 2. Discovery and elimination of wage inequities. 3. Establishment of sound wage foundation for incentive and bonus. 4. Maintenance of a consistent wage policy. 5. Creation of a method of job classification, so that management and union officials may deal with major and fundamental wage issues during negotiations and grievance meetings. 6. Installation of an effective means of wage control. 7. Collection of job facts to air (a) Selection of employees, (b) Promotion and transfer of employees, (c) Training of new workers, (d) Assignment of tasks to jobs, (e)Accident prevention, (f) Improving working conditions, (g) Administrative organization, and (h)Work simplification. A fourfold system of classifying evaluation systems is presented here. Two are described as nonquantitative and two as quantitative. 1. Non quantitative evaluation measures, a. The ranking system. b. The job classification system. 2. Quantitative evaluation measures. a. The point system. b. The factor comparison system.

3. Why is it important to handle grievances carefully?


What might happen if an organization does not provide some method by which a employee can voice his complaints and obtain a explanation? The employee will be unhappy, his productivity is impacted, he openly begins to share his discontent with not just his colleagues but also outsiders, friends, relatives, maybe even customers and vendors. Just as the employee has all the right to voice a grievance, as employer (or the management) owes it to the employee to respond suitably to the grievance. It is but commonsense that the resolution of a problem rests
60 | P a g e | S P R I N G 2 0 1 2

SEMESTER 1 - 511231186

on management. The earliest and clearest opportunity for issue resolution is found at the first stage, before the grievance has left the jurisdiction of the manager. For this reason, many firms have specifically trained their managers on how to handle a grievance or complaint properly. If the dispute or grievance constitutes a managerial problem it can often be resolved by the manager himself with the help of the HR team. The following steps discuss how a grievance can be redressed: 1. Receiving the grievance: The manner and attitude with which the manager receives the complaint of grievance is important. The basic premise is that the manager should at the outset assume that the employee is fair in presenting his/her opinion/complaint. The complaint should not be prejudged on the basis of past experience with this or other employees. When a employee approaches the manager with a issue the manager needs to make himself available to listen it all out and provide him/her the undivided attention. Research confirms that managers who were more task-oriented, as contrasted with managers who were more people-oriented, tended to experience a significantly higher number of grievances being filed in their units. 2. Reviewing the grievance: Once a complaint is received all facts supporting the issue needs to be gathered. Proper record keeping such as performance ratings, job ratings, attending records, and suggestions are reviewed. In addition, with the increasingly legal implications of modern labor-management relations, the manager should keep records on each particular grievance. All action taken, discussions with the employee, summary and what is agreed to all of it needs to be recorded. 3. Analysis and decision: With the problem defined and the facts in hand, the manager must now analyze and evaluate them, and come to some decision. It is important for the manager to involve others in the process to ensure that it is fair and is the best solution. The manager must include the views of his own manager as he might not be aware of all the implications of the problem and its resolution. Involving HR too is a recommended process in all organizations. HR can then seek finance or legal counsel if required, before any decision is taken. All involved in the decision making process need to be aware that the decision may create an undesirable precedence within the department as well as the company. 4. Response: Often it might not be possible to provide a positive resolution to the problem. If the solution decided is adverse to the employees views, attention needs to be given to the method of communication. Employees dislike managers who will take no stand, good or bad. Clearly communicating the message and sharing as much information as possible about the decision making process helps in establishing credibility to the process used to make the decision. The manager can also invite HR or his manager to sit-in on the conversation with the employee. As far as possible this should happen in a face-to-face meeting. In the event an employee wishes to take the appeal beyond to the next stage of the procedure he must be allowed to do so. The manager must have the opportunity to explain his decision to the other members so they can take a well-informed decision. 5. Follow up: The objective of the grievance procedure is to resolve a disagreement between an employee and the organization. Open communication is important for this process. The purpose of phase is to determine whether the employee feels that the problem has been sufficiently redressed. If follow up reveals that the case has been handled unsatisfactorily, then redefinition
61 | P a g e | S P R I N G 2 0 1 2

SEMESTER 1 - 511231186

of the problem, further fact-finding, analysis, solution and follow up are required. At this stage the manager can step aside and allow someone else in a position of authority like the HR or the managers manager to lead the process and close it.

4. How can we evaluate the effectiveness of training programs conducted in organizations?


Measuring the effectiveness of training programs consumes valuable time and resources. Many training programs fail to deliver the expected organizational benefits. Having a wellstructured measuring system in place can help you determine where the problem lies. On a positive note, being able to demonstrate a real and significant benefit to your organization from the training you provide can help you gain more resources from important decision-makers. Consider also that the business environment is not standing still. Your competitors, technology, legislation and regulations are constantly changing. What was a successful training program yesterday may not be a cost-effective program tomorrow. Being able to measure results will help you adapt to such changing circumstances. The Kirkpatrick Model The most well-known and used model for measuring the effectiveness of training programs was developed by Donald Kirkpatrick in the late 1950s. It has since been adapted and modified by a number of writers; however, the basic structure has well stood the test of time. The basic structure of Kirkpatrick's four-level model is shown here. Level 4 - Results What measurable organizational benefits resulted from the training in terms such as productivity, efficiency and sales revenue? Level 3 - Behavior To what extent did participants change their behavior back in the workplace as a result of the training? Level 2 - Learning To what extent did participants improve knowledge and skills and change attitudes as a result of the training? Level 1 - Reaction To what extent did the participants find the training useful, challenging, well-structured, organized, and so on? An evaluation at each level answers whether a fundamental requirement of the training program was met. It's not that conducting an evaluation at one level is more important that another. All levels of evaluation are important. In fact, the Kirkpatrick model explains the usefulness of performing training evaluations at each level. Each level provides a diagnostic checkpoint for problems at the succeeding level. So, if participants did not learn (Level 2), participant reactions gathered at Level 1 (Reaction) will reveal the barriers to learning. Now moving up to the next level, if participants did not use the skills once back in the workplace (Level 3), perhaps they did not learn the required skills in the first place (Level 2).
62 | P a g e | S P R I N G 2 0 1 2

SEMESTER 1 - 511231186

The difficulty and cost of conducting an evaluation increases as you move up the levels. So, you will need to consider carefully what levels of evaluation you will conduct for which programs. You may decide to conduct Level 1 evaluations (Reaction) for all programs, Level 2 evaluations (Learning) for "hard-skills" programs only, Level 3 evaluations (Behavior) for strategic programs only and Level 4 evaluations (Results) for programs costing over $50,000. Above all else, before starting an evaluation, be crystal clear about your purpose in conducting the evaluation. Using the Kirkpatrick Model Level 1 (Reaction) completed participant feedback questionnaire informal comments from participants focus group sessions with participants Level 2 (Learning) pre- and post-test scores on-the-job assessments supervisor reports Level 3 (Behavior) completed self-assessment questionnaire on-the-job observation reports from customers, peers and participant's manager Level 4 (Results) financial reports quality inspections interview with sales manager

5. What are the objectives of human relations?


Human Relations in Management is a process that brings employees into contact with and causes them to be influenced by their leaders, their jobs, and other aspects of the organizations which they work in. It includes everything at the workplace that influences the behavior of employees: their relationships with their mangers, with other employees, the kind of work they do, and the impact on them of the practices of the organization as a whole. As discussed earlier this knowledge of the employees expectations are critical for the achievement of the business results of revenue, profit, growth, market share. In the best possible way, "human relations refer to the interaction of people in all walks of life-in schools, homes, business and government." Huneryager and Heckmann define the expression thus: Human relations are a systematic, developing body of knowledge devoted to explaining the behavior of individuals in the working organization." In the opinion of McFarland, "Human Relations is the study and practice of utilizing human resources through knowledge and through an understanding of the activities, attitudes, sentiments, and inter-relationships of people at work."When applied to a business or an industry, it refers to every dimension of interaction that happens between employee and employee, employee and manager, teams and managers, employee and customer and employee and vendor (one who provides a support or a service). It is therefore a key objective of every organization to ensure that all dimensions of the human relations are redressed appropriately to ensure positive human relations. When a relationship exists in an organization, it is referred to as
63 | P a g e | S P R I N G 2 0 1 2

SEMESTER 1 - 511231186

employee-human relations; and when it exists outside it, it is known as public human relations. Organizations need to pay due attention to not just the way it treats employees but also the methods and processes in place to recognize and reward teams of employees depending on what they achieve and deliver for the organization. This calls for another dimension in managing human relations that focus on the employee as a social animal. To place it in perspective, the human relations focus of an organization needs to have programs in place to take care of: a) Employee needs b) Team needs c) Customer/vendor needs. Objectives of Human Resources Management Objectives are pre-determined goals to which individual or group activity in an organization is directed. Objectives of personnel management are influenced by organizational objectives and individual and social goals. The objectives of HRM may be as follows To create and utilize an able and motivated workforce, to accomplish the basic organizational goals. To establish and maintain organizational structure and desirable working relationships among all the members of the organization. To secure the integration of individual or groups within the organization by co-ordination of the individual and group goals with those of the organization. To create facilities and opportunities for individual or group development so as to match it with the growth of the organization. To attain an effective utilization of human resources in the achievement of organizational goals. To identify and satisfy individual and group needs by providing adequate and equitable wages, incentives, employee benefits and social security and measures for challenging work, prestige, recognition, security, status. To maintain high employees morale and human relations by sustaining and improving the various conditions and facilities. To strengthen and appreciate the human assets continuously by providing training and development programs To consider and contribute to the minimization of socio-economic evils such as unemployment, under-employment, inequalities in the distribution of income and wealth and to improve the welfare of the society by providing employment opportunities to women and disadvantaged sections of the society. To provide an opportunity for expression and voice management To provide fair, acceptable and efficient leadership To provide facilities and conditions of work and creation of favorable atmosphere for maintaining stability of employment.

64 | P a g e | S P R I N G 2 0 1 2

SEMESTER 1 - 511231186

6. Assume yourself as an HR Manager. You have been given the responsibility of promoting the rightful employees. For this, performance appraisal of the employees must be carried out. What appraisal method would you choose? Justify.
A performance appraisal, employee appraisal, performance review, or (career) development discussion is a method by which the job performance of an employee is evaluated (generally in terms of quality, quantity, cost, and time) typically by the corresponding manager or supervisor. A performance appraisal is a part of guiding and managing career development. It is the process of obtaining, analyzing, and recording information about the relative worth of an employee to the organization. Performance appraisal is an analysis of an employees recent successes and failures, personal strengths and weaknesses, and suitability for promotion or further training. It is also the judgment of an employees performance in a job based on considerations other than productivity alone. Aims Generally, the aims of a performance appraisal are to: Give employees feedback on performance Identify employee training needs Document criteria used to allocate organizational rewards Form a basis for personnel decisions: salary increases, promotions, disciplinary actions, bonuses, etc. Provide the opportunity for organizational diagnosis and development Facilitate communication between employee and employer Validate selection techniques and human resource policies to meet federal Equal employment Opportunity requirements. To improve performance through counseling, coaching and development.

Methods A common approach to assessing performance is to use a numerical or scalar rating system whereby managers are asked to score an individual against a number of objectives/attributes. In some companies, employees receive assessments from their manager, peers, subordinates, and customers, while also performing a self assessment. This is known as a 360-degree appraisal and forms good communication patterns. The most popular methods used in the performance appraisal process include the following: Management by objectives 360-degree appraisal Behavioral observation scale Behaviorally anchored rating scales Trait-based systems, which rely on factors such as integrity and conscientiousness, are also used by businesses but have been replaced primarily by more objective and results-oriented methods. The scientific literature on the subject provides evidence that assessing employees on factors such as these should be avoided.
65 | P a g e | S P R I N G 2 0 1 2

SEMESTER 1 - 511231186

The reasons for this are twofold: Trait-based systems are by definition based on personality traits and as such may not be related directly to successful job performance. In addition, personality dimensions tend to be static, and while an employee can change a behavior they cannot change their personality. For example, a person who lacks integrity may stop lying to a manager because they have been caught, but they still have low integrity and are likely to lie again when the threat of being caught is gone. Trait-based systems, because they are vague, are more easily influenced by office politics, causing them to be less reliable as a source of information on an employees true performance. The vagueness of these instruments allows managers to assess the employee based upon subjective feelings instead of objective observations about how the employee has performed his or her specific duties. These systems are also more likely to leave a company open to discrimination claims because a manager can make biased decisions without having to back them up with specific behavioral information. People differ in their abilities and their aptitudes. There is always some difference between the quality and quantity of the same work on the same job being done by two different people. Performance appraisals of Employees are necessary to understand each employees abilities, competencies and relative merit and worth for the organization. Performance appraisal rates the employees in terms of their performance. Performance appraisals are widely used in the society. The history can be dated back to the 20th century and then to the second world war when the merit rating was used for the first time. An employer evaluating their employees is a very old concept. It is an indispensable part of performance measurement. Performance appraisal is necessary to measure the performance of the employees and the organization to check the progress towards the desired goals and aims. The latest mantra being followed by organizations across the world being get paid according to what you contribute the focus of the organizations is turning to performance management and specifically to individual performance. Performance appraisal helps to rate the performance of the employees and evaluate their contribution towards the organizational goals. If the process of performance appraisals is formal and properly structured, it helps the employees to clearly understand their roles and responsibilities and give direction to the individuals performance. It helps to align the individual performances with the organizational goals and also review their performance. Performance appraisal takes into account the past performance of the employees and focuses on the improvement of the future performance of the employees.

66 | P a g e | S P R I N G 2 0 1 2

SEMESTER 1 - 511231186

MB0043 Human Resource Management - (Assignment Set- 2)


1. State the major career development activities found in organizations.
A broad view of career is defined as an individually perceived sequence of attitudes and behaviour work-related activities and experiences over the span of a persons life. In normal parlance the term career has both an internal and an external focus. An internal focus and refers to the way an individual views his/her career and the external or objective focus and refers to the actual series of job positions held by the individual. The dynamics of career development in an organizational context has two dimensions: How individuals plan and implement their own career goals (career planning), and How organizations design and implement their career development programs (career management). Career planning is a deliberate attempt by an individual to become more aware of their skills, interests, values, opportunities and constraints. It requires an individual thinking to identify career-related goals and establishing plans towards achieving those goals. Often it is a self-driven process, which every professional (irrespective of the nature of employment) spends some time to dwell on and discuss it with peers or superiors and frame it. It is also viewed from time to time that the individual looks for possible new career options. Having a career plan builds a commitment towards achieving it and is viewed as an excellent personal goal-setting exercise for self motivation. Career management is considered to be an organizational process that involves preparing, implementing and monitoring career plans undertaken by an individual alone or within the organizations career systems. Organizations establish policies that provide for multiple career path options that an employee can choose from and pursue. This is supported with a lot of training and development activities that are agreed to with the manager and planned carefully and executed. A variety of career development activities and tools exists for use in organizations. HR managers should be familiar with these components because the managers often serve as internal consultants responsible for designing the career development systems. Some of the activities described are individual career planning tools and others are commonly used for organizational career management. In general the most effective career development programs will use both types of activities. A variety of career development activates are available for use. Some of the more popular ones include: 1. Self assessment tools:- these are usually technology enabled on-line (on the corporate intranet) tools that form part of the performance appraisal system and allow the individual to identify areas of strengths and parallelly identify career paths that would leverage these strengths the best. E.g. Career Planning Workbooks, Career Workshops hosted by the organizations from time to time.

67 | P a g e | S P R I N G 2 0 1 2

SEMESTER 1 - 511231186

2. What are the major problems faced in benefits management?


One of the main problems faced by management in organizations is the need to maintain a competitive advantage. With the global economic climate it has been difficult for companies and organizations to keep up with their rivals, let alone better them. Corporate culture is one responsibility that management need to adhere to because it is vital when you want to succeed in business. It creates a sense of innovation and productivity as opposed to a more negative culture which may stifle employees and detrimentally affect job gratification. In addition, managers have a huge responsibility for guiding the organization in the right direction as they are the prime decision makers. Managers have to make the plans and organize their employees and resources in order to put the organization in a direction that will grant them success. Most modern management take on a strategic management style which initially states the main aim of the particular mission which follows by the processes that will be carried out on a day-to-day basis. Moreover, one of the most important aspects that a good manager requires is solid communication skills. It is very likely in business that you are going to come into contact with conflicting styles of communication, however it is the manager's job to be aware of this as well as their own style. They need to be able to enforce their own style of communication while being able to adapt to others, especially if they are consulting with other organisations and companies. Some of the basic problems faced by management are 1: How to produce a qualitative product: This is the first problem faced by management that what is to produce, how much to produce and where to be produce. And the organization has to decide either they have to produce different products or to emphasis on one product. 2: How to deal the labour union: The labour is the group of people working for the betterment of the employees working in the organization. The management has to decide that how to full fill the demands of the labour union in respect of salaries, bonuses, insurance, medical allowances, fringe benefits etc.

3: How to compete in the market: Various decisions for example how to charge the price, how to place the product, how to promote the product has to be taken by the management and they try to solve these problems in a best manner. 4: How to utilize the organization resources: The management made various decisions about the organization resources that is man, money, material, machinery, market and methodology.
68 | P a g e | S P R I N G 2 0 1 2

SEMESTER 1 - 511231186

5: To avoid stick out situation: Stock out situation is that situation when the customer demands for the product and the company has no product at that time. The management has to decide how to tackle this problem.

3. What are the factors that impact recruitment in organizations?


The recruitment function of the organisations is affected and governed by a mix of various internal and external forces. The internal forces or factors are the factors that can be controlled by the organisation. And the external factors are those factors which cannot be controlled by the organisation. The internal and external forces affecting recruitment function of an organisation are:

4. Explain the general procedures followed in the case of a disciplinary action.


In general, discipline for employees is applied in progressive steps as follows: 1. Oral warning 2. Written warning 3. Final written warning, suspension and/or disciplinary probation 4. Termination of employment

69 | P a g e | S P R I N G 2 0 1 2

SEMESTER 1 - 511231186

This policy is not contractual in nature and does not constitute an agreement that any particular procedure or rule will apply. The nature of the offense and the particular circumstances determine whether or not all or any of the steps in the above sequence are followed. Disciplinary steps may be omitted, accelerated, or repeated as the University determines appropriate. The purpose of disciplinary measures short of termination is corrective, to encourage employees to improve their conduct or performance so that they may continue their employment with the University. The University expects all employees to behave in a mature and responsible manner and to perform their jobs conscientiously, without the need of disciplinary action. These corrective disciplinary measures will not apply in the event of any offense that the University determines to warrant immediate termination of employment or in other circumstances when the University determines that corrective measures would be ineffectual or otherwise inappropriate. Oral Warnings: If an employee is given an oral warning the employee is informed of the warning by his/her manager. The warning is also recorded by the manager in writing and the record is placed in the employee's personnel file. Written Warnings: If an employee is issued a written warning or a final written warning, the manager will meet with the employee to discuss the disciplinary action and the employee will be asked to sign the warning. The employee's signature is only an acknowledgment that the employee has been informed of the warning; it does not indicate agreement with the warning. All written warnings are retained in the employee's file. Disciplinary Probation: An employee may be placed on disciplinary probation for unsatisfactory performance or conduct. An employee placed on disciplinary probation will be given a written notice of probation, which generally provides an explanation of the reason for the action, the length of the probationary period and a plan of corrective action to be successfully completed during the period. The employee's manager will meet with the employee to discuss the terms of the disciplinary probation, and the employee will be asked to sign the notice of probation as an acknowledgement that the employee has been informed of the action. A copy of the notice is retained in the employee's personnel file. At the conclusion of the probationary period, and from time to time during the period, as appropriate, the employee's manager will meet with the employee to review his/her progress. An employee on disciplinary probation who does not show satisfactory improvement is subject to further disciplinary action at any time, up to and including termination of employment. Investigative Suspension: A suspension from work may be appropriate when circumstances require an investigation and it does not appear practical or desirable or in the best interests of the University or of the employee for the employee to remain at work during that investigation. An investigative suspension is not itself a disciplinary measure. If, at the conclusion of the
70 | P a g e | S P R I N G 2 0 1 2

SEMESTER 1 - 511231186

investigation, the investigative suspension is not converted to a disciplinary suspension or discharge, the employee will be reinstated and will be paid at his/her normal straight-time rate for all regularly scheduled work missed during the suspension. Disciplinary Suspension: A disciplinary suspension is a suspension from work without pay for one or more days for a repeated or serious infraction of University rules or policies. A record of the suspension is retained in the employee's personnel file. Misconduct during the Probation Period: The disciplinary action policy is not normally applied to new employees during the probationary period. Unsatisfactory performance or any infraction of University rules or policies or other misconduct during this period may result in immediate termination of employment. Probationary employees are not eligible to use the grievance procedure.

5. Trace out the growth of Human Resource Management in India.


Contrary to these forces, in INDIA the owner manager/ government/public sector manager was an industry icon and a national hero of sorts. The personnel management practices were dominant of the brick- and-motor industry. Though the approach it was largely welfare oriented and reactive in nature it served effectively for the large PSU organizations that built the countrys foundation. A large part of the workforce was migrant from the rural parts of the country and armed with educational qualifications that served as their passport to a secure future. Many left the country to study and pursue career overseas. This phenomenon was called brain drain and happened during 1990 through 2005. The best of the Indian talent left the country. Per present statistics there are close to 1.7 million people of Indian origin in AMERICA according to the US census bureau. The INDIAN AMERICAN median family is $60,093 as against the national median family income of $38,885 left the country, the estimated buying power of the INDIAN AMERICANS in the United States is around US $20 billion, the high income clearly reflects the advanced educational levels achieved by Indian abroad. Its only in the past 10-12 years with the immense growth on account of the IT industry that winds of change began to blow. It was largely the advent of the information technology era in INDIA that brought with it the western management practices. MNCS started up their operations in INDIA the FDI (FOREIGN DIRECT INVESTMENT) went up steeply as the world saw the potential in the country human resources. In INDIA became a preferred location for MNCs primarily from the USA followed by others developed countries. It was natural for the MNCs setting up operations in INDIA to establish their existing and proven
71 | P a g e | S P R I N G 2 0 1 2

SEMESTER 1 - 511231186

management practices that were successfully working in the organization back home. It also helped them manage their business similarly. This gave birth to a new generation of management as well as HRMpractices. New hiring methods new ways of paying salaries, new employment terms and most importantly increased focus on individual performance and outcome. There was emphasis on deliverables and linking individual and team performance to business results and success. Given the highly educated workforce there was a de-emphasis in the role of the trade unions, the era of the trade unions dominance give way to the new order of individual negotiated salaries and terms and clearly performance linked assessment system. Another transformation that the Indian workplace witnessed was focused on the ethics and ethical practices in doing business it was only fair to except that with the weak legal system, it needed the support of the government policies and the corporate policies to beat the corruption that existed .This has significantly contributed to INDIA emerging as a preferred destination for doing business. All of this yielded to give the way to the birth of the professional manager. Professional manager today are a critical and essential part of the INDIAN corporate. The professional manager brought about a shift in the culture from a highly author it Arian approach of getting work done to more collaborative and participative approach. In the traditional Indian culture where the child is brought up to dependents on parents and superiors these shift was break through and took it time to manifest. The entrepreneurs who earlier operated in a secure, sheltered market and hardly face challenges , were challenged by the globalization that swept in with the liberalization policies and measure brought in by the Indian employee and his manager evolved. Together they stopped up to face the challenges head- on and to win not only in INDIA but also globally. The levers of a) low cost b) highly skilled c) English as the medium of education and it being the corporate language that enabled the flow of global business to INDIA? Hence human relations movement in INDIA has evolved very differently as compared to the developed economies of the USA and the UK. What is currently acting as a limitation is the enhanced awareness on the need for research based HRM practices. While there is a lot of work happening in education system to promote this.

6. Assume yourself as an HR Manager of a publishing house. You find that the morale of the employees is generally low. What steps would you take to improve employee morale?
The motivation techniques may be divided into two parts [A] that is to be done; and [B] How and why what is done. The former are steps in motivation and the later are rules governing the steps. Both are performed simultaneously.

72 | P a g e | S P R I N G 2 0 1 2

SEMESTER 1 - 511231186

These are listed below: a) Size up situation requiring motivation: The first step of motivation is to make sure of motivational needs. Every employee needs motivation; however, all people do not react exactly as the same way to the same stimuli. Keeping this in mind executive shall size up how much and what kind of motivation is needed. b) Prepare a set of motivational tools: An executive from his personnel experience should prepare a list of what devices are likely to work with what type of people and under what circumstances. c) Selecting and applying motivator: Proper application of motivational plan is important. This involves selection of the appropriate technique, method of application and the timing and location of applications. d) Follow up the results of applications: The last stage of motivation is to follow-up the results of the application of the plan. The primary objective is to ascertain if an employee has been motivated or not. If not some other technique should be tried. Rules of motivating: The motivation manager must be guided with some fundamental rules which should be based on the following principles. a. Self interest and motivation: Motivation is mainly built on selfishness. Psychologically speaking, selfishness is a part and parcel of life. To deny this is to build the theory on unrealistic foundation. To seek some other basis of motivation would be to ignore the real nature of man. The aim should be to learn more about selfishness. b. Attainability: Motivation must be establish attainable goals. This does not mean that the goal is realized at once. This may take years to attain. But it must be within reach. Eight ways to motivate plant employees: The following recommendations are for improving the motivation of employees in the routine jobs. a. Provide assembly line employees with more than minimum training. This would result in greater personnel involvement in the job. b. Create sub goals to measure accomplishment. A sense of completion is important for motivation. They are likely to be more interested in the work which will reduce monotony and mental fatigue. c. Provide regular feedback on performance. Studies show that people work better when they receive positive feedback. d. Maintain a neat and orderly work area. If the foreman does not care abouthousekeeping then employees may feel that they also need not care about it and this attitude may affect quality of work.

73 | P a g e | S P R I N G 2 0 1 2

SEMESTER 1 - 511231186

e. Arrange work situations so that conversation between employees is either easy or possible. Experience workers may to their job with little attention to the task.Conversation my reduce monotony and thus fatigue. f. Increase the number of operations performed by one employee. This can be done by simplification of manual operations. It offers several advantages: The risk of errors reduced; Management can hire employees at lower wages; Training costs are minimized.

g. Structure jobs, so that workers can at least occasionally move about the work area. Besides job rotation, there are other ways to provide for physical movement like stetting employees secure their own tools etc. h. Explore ways to assign greater personnel responsibility. Increased responsibility means greater self esteem and greater job meaningfulness. One way to enlarge responsibility is to let the employee inspect his own work.

_______________________________________________________________________________ ________________ THANK YOU______________________________________________________ _______________________________________________________________________________ _____________________________ SHIBIL JALEEL _________________________________

74 | P a g e | S P R I N G 2 0 1 2

Das könnte Ihnen auch gefallen